Urinary unit 10 --- hematological unit 11---105

Réussis tes devoirs et examens dès maintenant avec Quizwiz!

Nephrotic Syndrome-

*Not due to inflammation* and you do not see red blood in urine** Glomerular tissue damage significant protein loss in urine

Knowing that a variety of problems can contribute to the development of anemia, you as patients about any history of:

-Gastric surgery -Kidney disease -Poor nutrition -Bleeding episodes -Alcohol abuse

a variety of problems can contribute to the development of anemia,

-Gastric surgery -Kidney disease -Poor nutrition -Bleeding episodes -Alcohol abuse

A comprehensive care plan has been developed for Mrs. Byrd. Which of the following criteria would indicate successful outcomes for Mrs. Byrd?

-Patient reports feeling comfortable, neither hot nor cold -A hemoglobin between 12.0 and 16.0 g/dL -Oral cavity clean and intact -Participates in activities of daily living without assistance -Patient reports no injuries -Patient reports absence of dizziness

Urinary Incontinence Contributing factors:

-Pelvic muscle relaxation -Impaired neural control -Bladder problems Incontinence happens if the bladder muscle suddenly contracts (spasms) or sphincter muscles are not strong enough to hold the urine back. Brain may also tell the brain to empty, when it is not full *Not a normal part of aging Age-related changes contributes to UI • Decreased bladder capacity • Involuntary muscle contractions **Decreased estrogen • Constipation: can act as an obstruction making us hold on to urine • Low fluid intake: may not want to drink a lot of water if always leaking • Chronic (degenerative) diseases: difficulty getting out of chair or bed • Medications ***Incontinence increases risks for falls, fractures, pressure ulcers, UTIS, depression and caregiver burn out. -Most falls occur in bathroom

Comprehensive instructions for patients as they prepare to go home include which of the following?

-Perform mouth care frequently -Eat leafy green vegetables -Wear or carry warm clothing

Anemia home care instructions

-Perform mouth care frequently-Eat leafy green vegetables-Wear or carry warm clothing

Surgery: Lithotripsy

-Shock waves break up stones: done as outpatient procedure

Glomerulonephritis A result of thistype of bacteria:

-​glomerulonephritis usually follows an infection with group Abeta-hemolytic Streptococcus, such as strepthroat.

NephroticSyndrome Two key manifestations for this disease are:

-​proteinuria​ -​low serum albumin levels -​ high blood lipids and edema( Edema may be severe, affecting the face and periorbital area as well as dependent tissues)

Urinary Retention Two Types:

1) Obstruction: Urine cannot flow freely through the urinary tract Common causes: Cancer (tumor), kidney or bladder stones or enlarged prostate in men 2) Non-obstruction; weak bladder muscle and nerve problems that interfere with signals between the brain and the bladder Common causes: stroke, vaginal child birth, pelvic injury or trauma, impaired muscle or nerve function due to medication or anesthesia and accidents that involve the brain or spinal cord

Key teaching concepts to prevent a recurrence ofUTI:

1)Empty bladder every 2-4 hrs while awake, avoid voluntary urinary retention 2) maintain 2 to2.5 quarts or 8to 10 glasses of fluid per day. 3)complete prescribed treatment. 4) drink 2 or more glasses of cranberry juice and consume blueberries regularly. and wipe front to back avoid sugar drinks/foods, caffeine, alcohol

Urinary What is the difference between the 2 types of BPH meds?

1)alpha1 blockers like terazosin (hytrin) and tamsulosin (flomax) relax smooth muscles and reduce obstruction improving urinary flow 2) androgen inhibitor, finasteride (Proscar) inhibits conversion of testosterone 1 to DHT in prostate, causing shrinkage. (can cause impotence).

A 19-year-old patient is admitted in sickle cell crisis. The FIRST nursing action would be to: 1. Administer pain medications 2. obtain blood samples for analysis. 3. administer antibiotics. 4. Insert a Foley catheter.

1. Administer pain medications

Glomerulonephritis Effective treatment would result in:

1. ​identifying and treating the underlying disease process 2.pre serving kidney function Lower BP less edema

Kidney Failure Name 2 criteria for the diagnosis of acute kidney injury:

1.SERUM CREATININE (SCr) AND GLOMERULAR FILTRATION RATE (GFR):S​Cr increased 2 times or GFR >50% 2.URINE OUTPUT: <​ 5mL/kg/hr for 12hr 1. oliguria 2. high BUN and/or creatinine 3. Decrease GFR

What needs to be identified in a U/A for a diagnosis of cystitis/pyelonephritis?

1.blood cells 2.bacteria "A urinalysis is ordered to identify blood cells and bacteria in the urine. A urine culture and sensitivity may also be done to identify the causative organism. A complete blood count (CBC)with differential is done to assess for systemic responses to infection."

Urinary What is each port of a 3 way catheter used for:

1.irrigation 2.Balloon inflation 3.Urine Drainage

Kidney Failure To assess for a proper functioning fistula in a hemodialysis patient, assess for:

1.​ A palpable pulsation 2. ​An audible bruit (murmur)

UrinaryName 2 potential complications for someone with BPH?

1.​ hydronephrosis ( distension of kidney with urine) 2.urinary tract infection

Urinary Calculi Nursing interventions when stones calculi are suspected:

1.​Administer analgesia as ordered. Pain control is vital to reduce anxiety, stress and to facilitate healing. 2.​Unless contraindicated increases fluid intake and encourages ambulation. increased fluids and ambulation increase urinary output help to move the stone through the ureter and decrease pain. 3.​Strain all urine for stones, saving recovered stones for laboratory analysis. analysis of stones helps direct treatment to prevent further stone formation

Glomerulonephritis Since I will have sodium and water retention, I expect to have these 2 symptoms:

1.​Edema 2.​Hypertension

Glomerulonephritis 2 key compounds will appear in my urine

1.​Protein uria (protein in urine) leads to hypoalbuminemia (low albumin in blood) 2. azotemia (increased levels of nitrogenous wastes, urea, creatinine) and hematuria

Collaborative Care Pyelonephritis:

1st: Urinalysis by clean catch: have patient use antiseptic wipes front to back, start stream, stop, then into cup -wait for culture and sensitivity results: start patient on broad spectrum antibiotic CBC: to check for systemic response- Look at WBC >11,000 IVP: X-ray with dye to outline the kidneys, ureters and bladder: diagnosis why there is blood in urine or pain on sides or lower back. Can also detect kidney stones, enlarged prostate, tumors, scarring from UTI's, and birth defects of urinary tract system Voiding cystourethrogram: patient lies on back, insert dye, pictures are being taken as patient is voiding

Nephrotic Syndrome Classification of medications to decrease edema and protein loss:

1​.angiotensin - converting enzyme(ACE) inhibitors reduce BP and the amount of protein released in urine 2.Anti hypertensive prescribed to control BP

iron-deficiency anemia. Which of the following questions is pertinent to her diagnosis?

"Is your tongue sore?"

Patient has iron-deficiency anemia. Which of the following questions is pertinent to her diagnosis? You expect that the physician will prescribe which of the following

"Is your tongue sore?" another manifestation is spooning of the nails Feosol (ferrous sulfate)=side effects=Nausea, constipation (Increase her fluid intake)

Which statement regarding continuous ambulatory peritoneal dialysis (CAPD) is of highest priority when teaching a patient new to this procedure?

"It is essential that you maintain aseptic technique to prevent peritonitis." Peritonitis is a potentially fatal complication of peritoneal dialysis, and it is imperative to teach the patient methods to prevent it from occurring. Although the other teaching statements are accurate, they do not address the potential for mortality by peritonitis, making that nursing action the highest priority.

The nurse has provided nutritional teaching on foods high in folate to a client with folate deficiency related to malabsorption syndromes and poor nutrition. Which of the following foods, if chosen by the client, indicates that the client understands the teaching? a. Liver, dark green leafy vegetables b. Whole milk and eggs c. Potatoes and carrots d. Bread and fish

ANSWER: a. Liver, dark green leafy vegetables RATIONAL: Foods high in folate are liver, orange juice, cereals, whole grains, beans, nuts, and dark leafy vegetables like spinach

It is important to assess patients for signs and symptoms of iron overdose. These can include:

Abdominal pain

PATIENT TEACHING Prostate Surgery

Activity Healing requires 4 to 8 weeks. Avoid strenuous activity and heavy lifting. Do not drive for 2 weeks. Take long walks but take stairs slowly and carefully. Continue leg exercises (dor- siflexing the foot) to prevent blood clots in the legs. You can take showers but avoid tub baths while the catheter is in place (National Institutes of Health, 2012). Bleeding Bleeding may occur after a bowel movement, coughing, or increased exercise. If you notice blood in the urine, increase fluids and rest until urine is clear. If you have clots or are unable to urinate, call your doctor immediately. Avoid aspirin and non- steroidal anti-inflammatory drugs (NSAIDS) for at least 2 weeks. Bowel Movements Keep bowel movements regular and soft to avoid pressure on the prostate area. Eat fruits and other high-fiber foods, and take stool softeners as ordered. Diet Resume your normal diet. Increase fluids to 3 L (13 cups) daily. Avoid alcohol unless otherwise advised by your physician. Sexual Intercourse Do not have sex for 6 weeks after surgery to avoid bleeding. You may still have erections even with the catheter in place. When you resume sex, ejaculate may flow back into the blad- der; if this is so, you will express little or no semen. Urination After the catheter is removed, you may experience some burn- ing, stinging, or leakage for several weeks, and you may pass small blood clots occasionally. These symptoms disappear as the area heals. Use pads to control leakage. Work If work is not strenuous, you may return in 4 weeks. Otherwise, wait 6 to 8 weeks. Contact your doctor immediately if: You are unable to urinate. Bleeding is excessive or is not controlled by fluids and rest. You have chills and fever or severe abdominal pain. 1 Your scrotum becomes swollen and tender. 1 You have pain in one calf, chest pain, or difficulty breathing.

Following an intravenous pyelogram (IVP), the nurse encourages fluid intake, recognizing that this is necessary to prevent which renal complication?

Acute kidney injury The dye used for an IVP is nephrotoxic. If not excreted fully and quickly will place a patient in AKI.

Urinary Calculi Priority NANDA nursing diagnosis for kidney stones:

Acute pain

Which nursing diagnosis is a priority in the care of a patient with renal calculi?

Acute pain Urinary stones are associated with severe abdominal or flank pain. Deficient fluid volume is unlikely to result from urinary stones, whereas constipation is more likely to be an indirect consequence rather than a primary clinical manifestation of the problem. The presence of pain supersedes powerlessness as an immediate focus of nursing care.

Nursing Care:

Address pain: Analgesia and fluids= tamsulosin Goal: to regain normal urinary elimination pattern

Which of the following nursing interventions should be included in the post-lithotripsy plan of care for patients renal colic and nephrolithiasis who has lithotripsy.

Administer the prescribed analgesic, strain urine, encourage oral fluids and frequent vital signs.

How are iron supplements given to a child?

Administration of Iron o administer iron in one to three divided doses between meals. o give with vitamin C rich fluids such as orange juice. Vitamin C increases the absorption of iron by the body o administer iron through a straw or medicine dropper placed at the back of the mouth, away from the teeth. Brush or wipe off teeth because oral iron can stain the teeth. o recognize that iron supplements causes black, tarry stools. instruct the parents to monitor for and expect to see black, tarry stools. iron therapy causes black, tarry stools. if absent, it may indicate lack of adherence to therapy. o avoid administration of iron with milk or formula and cereal because iron binds, with calcium, thus impeding absorption. (Mckinney pg 1121-1122) If given iron elixir (liquid), then you need to take it with a straw and wipe teething because of its staining effect.

A patient is complaining of foul smelling urine and burning on urination. The nurse suspects a UTI and obtains a urine analysis. What part of the analysis should be reported to the physician if it is positive for UTI?

After obtaining a urine specimen and analysis the nurse would report presence of blood cells and bacteria in the urine showing infection. The nurse would report what the volume of bacteria and blood cells that were found was. A urine culture may also be done to identify a specific organism. (Burke 746).

Since patient is receiving morphine, which of the following assessments are relevant? patient comments that the morphine relieves some, but not all, of his pain. Which of the following nursing interventions could be used to further facilitate pain relief? renal colic and nephrolithiasis who has lithotripsy.

Allergies, BP and respirations Moist heat to the lower pelvis.

A diabetic patient with nephropathy is undergoing several diagnostic tests with contrast. What teaching needs to be done post procedure? What should be avoided?

Among the diagnostic tests used for a patient with diabetic nephropathy, a Intravenous pyelography (IVP) or a Retrograde pyelography may be done. Both of these diagnostic tests involve the use of "contrast" and x-rays to evaluate the urinary tract. The contrast medium is injected intravenously in IVP. Its filtered from the blood by the kidneys, allowing x rays to show the contrast entering the kidney pelvis, flowing through the ureters, and into the bladder. This allows evaluation of renal function and the position, size, shape, and structure of the of urinary tract organs. Retrograde pyelography differs from IVP by which the contrast media is instilled into the collecting=ng system of the urinary tract. (Burke P.733) Patient teaching for post-procedure: o Immediately notify the HCP if you develop a rash, difficulty breathing, rapid heart rate, or hives. o Increase fluid intake after the test is completed.(help flush out the contrast) o Listed below are things to avoid specifically after kidney scans or biopsy.. o Avoid coughing for 24-hours after the procedure. Avid strenuous activity such as heavy lifting for about 2 weeks after the procedure. Lastly, Report symptoms such as flank, back, shoulder, or abdominal pain or light-headedness to your doctor because they may indicate a complication. (Burke p.752)

The nurse is caring for a patient with a history of fatigue, dyspnea and dark stools. The patient states that his stool are very smelly. The patient's CBC reveals a HGB of 7g/dL. Based on the patient's history and condition, the nurse expects to prepare for which procedure? A) Bone marrow aspiration B) Comprehensive dietary inventory C) Computed tomography (CT) scan D) Esophagogastroduodenoscopy (EGD)

An EGD would be needed to check for a possible GI bleed

The nurse is checking lab values on a patient who has just had a hemodialysis treatment. What would indicate to the nurse that the patient's renal dialysis was ineffective?

An increase in serum creatinine Patients with chronic kidney disease will have an elevated serum creatinine. Dialysis will excrete creatinine and lower the level.

What is the relationship between anemia and chronic kidney disease?

Anemia is a condition in which the body has fewer red blood cells than it usually has. Red blood cells or erythrocytes are responsible for carrying oxygen throughout the body. With anemia, red blood cells carry less O2 to the tissues, particularly the heart and brain and as a result, they may not function as well as they should. Patients with chronic kidney disease (CKD) often have anemia. The healthy kidney produces the correct amount of a hormone called erythropoietin (EPO) which triggers the bone marrow to make more erythrocytes. In patients with CKD, the kidneys do not make enough EPO which in turn does not relay the correct information to the bone marrow and an insufficient amount of red blood cells are produced. Another relationship between the two could be the excessive use of dialysis in end-stage CKD in which blood is lost and could add to the total loss of erythrocytes. (Nursing Central, Burke, p. 766-774) When you see that a patient has low RBC, H & H, check to see their kidney function (serum creatinine), many times it's due to CKD or acute kidney injury.

How does a high respiratory rate (28 breaths/min) indicate anemia?

Anemia reduces the oxygen-carrying capacity of the blood. As tissue oxygenation decreases, the body attempts to restore adequate oxygen delivery. The heart and respiratory rates increase as a result. Burke p. 478 Approximately 97% of the oxygen in the blood is carried attached to the hemoglobin molecule in RBCs. Anemia, whether due to decreased RBCs or hemoglobin concentration, reduces the oxygen-carrying capacity of the blood, leading to tissue hypoxia. As tissue oxygenation decreases, the body attempts to restore adequate oxygen delivery. The heart and respiratory rates increase. Blood is redistributed to vital organs, causing pallor of the skin, mucous membranes, nail beds, and conjunctiva. Tissue hypoxia may cause angina, fatigue, dyspnea on exertion and night cramps. The kidneys release increased amounts of erythropoietin, which stimulates the bone marrow, causing bone pain. Poor oxygen to the brain can cause headache, dizziness, and dim vision. The severity of the manifestations of anemia depends on the cause and severity of the disorder.

A nurse is using the Z-track method of IM injection to administer iron dextran to a client with iron deficiency anemia. Which techniques should the nurse use to give this injection? Select all that apply? 1. Confirm the clients identity before administering the iron dextran 2. Inject the iron dextran into the deltoid muscle 3. Change the needle after drawing up the iron dextran 4. Before inserting the needle, displace the skin laterally by pulling it away from the injection site. 5. Inject the iron dextran after aspirating for a blood return 6. After removing the needle, massage the injection site.

Answer: 1,3,4,5 Rationale: Before administering any medication, the nurse confirms the client's identity. After drawing up iron dextran, she removes the first needle and attaches a second needle to prevent tracking the medication through the subcutaneous tissue when the needle is inserted to administer the injection by Z-track method, the nurse first displaces the skin laterally by pulling it away from the injection site. The nurse should aspirate for a blood return before administering iron dextran; if no blood appears, the medication may be injected. Iron dextran should be administered into the large dorsogluteal muscle only. After injecting iron dextran, the nurse shouldnt massage the site because this could force the medication into the subcutaneous tissue.

The nurse is teaching a client who has iron deficiency anemia about foods she should include in the diet. The nurse determines that the client understands the dietary modifications if which items are selected from the menu? 1. Nuts and milk 2. Coffee and tea 3. Cooked rolled oats and fish 4. Oranges and dark green leafy vegetables

Answer: 4 Rationale: Dark green leafy vegetables are a good source of iron, and oranges are a good source of vitamin C, which enhances iron absorption. All other options are not food sources that are high in iron and vitamin C.Test-Taking Strategy: Focus on the subject, diet choices for a client with anemia. Think about the pathophysiology of anemia and determine that the client needs foods high in iron and recall that vitamin C enhances iron absorption. Use knowledge of foods high in iron and vitamin C. Remember that green leafy vegetables are high in iron and oranges are high in vitamin C.Level of Cognitive Ability: EvaluatingClient Needs: Physiological IntegrityIntegrated Process: Nursing Process—Evaluation Content Area: Foundations of Care: Therapeutic Diets Health Problem: Adult Health: Hematological: Anemias Priority Concepts: Client Education; Nutrition Reference: Lewis et al. (2017), p. 610.

Which of the following blood components are decreased in anemia? A. Erythrocytes B. Leukocytes C. Granulocytes D. Platelets

Answer: A, Anemia is defined as a decreased number of erythrocytes (red blood cells). Leukopenia is a decreased number of leukocytes (white blood cells). Thrombocytopenia is a decreased number of platelets. Lastly, granulocytopenia is a decreased number of granulocytes (a type of white blood cells)

Medications: UTI

Antibiotics: Fluoroquinolones Sulfonamides: (Bactrim) Assess for allergies (rash) Closely monitor if on oral hypoglycemia Drink 8 glasses of water daily Urinary Anti-Infectives: (Macrodantin) Often used with chronic UTI's Fluid intake of 1500 to 2000 mL/daily Urinary Analgesic: (Pyridium) Relieves pain, burning, frequency and urgency Does NOT treat infection Stains urine (and underwear) reddish orange

Urinary Incontinence Classification of medications to help my urge incontinence:

Anticholinergics Oxybutynin (ditropan) Tolterodine (detrol) Solifenacin (vesicare)

Patients with aplastic anemia are at risk for certain conditions. What is it and how can it be avoided?

Aplastic anemia is a disorder in which bone marrow ceases to produce RBCs. It can be caused by radiation, harmful chemicals, autoimmune disorders, chemotherapy, other medications, or viral infections including HIV, HepC, and mononucleosis, but often times the cause goes unknown. The most serious condition these patients are at risk for is heart failure, but they are also at risk for bleeding and infection because aplastic anemia results in decreased platelets and WBCs as well as decreased RBCs. Preventing heart failure begins with assessing for signs of it, including but not limited to dyspnea when lying down, increasing fatigue, shortness of breath, respiratory crackles, and peripheral edema. Treating the anemia itself can also prevent this complication. Treatment includes immunosuppressive therapy (if the anemia is autoimmune related) or androgens which can stimulate RBC production (Burke 482), as well as a stem cell transplant, blood transfusion, or ending the treatment (ie chemotherapy) which is causing bone marrow destruction. Aplastic anemia may be temporary (in the case of chemotherapy or even pregnancy) or permanent and the treatment is determined by the cause and severity. Aplastic anemia or bone marrow aplasia is a rare form of anemia that occurs in less than 20 thousand Americans each year. It is characterized by having low red and white blood cells as well as low platelets (pancytopenia). As we know, red blood cells are responsible for carrying oxygen throughout the body via hemoglobin, white blood cells are part of the immune system that attack foreign cells, and platelets help with blood clotting. All of these cells originate as stem cells produced in our bone marrow before growing into the specific cells the body requires.. With an auto-immune disease like aplastic anemia, the lymphoctyes attack and destroy these precious stem cells in the bone marrow, thus leading to low numbers of each in the bloodstream. The current treatment is using anti-immune drugs that kill off the immune system to stop it from attacking the body's cells and in doing so, sort of resets it like rebooting a computer. There has been success with this method while concurrently taking precautions that the patients are not exposed to harmful germs while their immune system is down. Some ways to avoid this condition would be to stay safe from hepatitis, HIV and Epstein-Barr viruses, make sure you're not exposed to toxins in pesticides and insecticides, limit radiation/chemotherapy if possible. Also know that some rheumatoid arthritic drugs and some antibiotics can cause this condition as well as an unknown cause which would be named idiopathic aplastic anemia. (Nursing Central, Burke, et al, p. 4778-482)

THE EFFECTS OF RENAL FAILURE ON BODY SYSTEMS*

BODY SYSTEMCardiovascular COMPLICATIONS ∙ Atherosclerosis ∙ Hypertension ∙ Congestive Heart Failure ∙ Pericarditis ∙ Arrhythmias EFFECTS ∙ Hyperlipoproteinemia ∙ Fluid overload ∙ Pericardial sac irritated by uremic toxins ∙ Electrolyte imbalance BODY SYSTEMEndocrine COMPLICATIONS ∙Prolonged half-life of insulin EFFECTS ∙ Decreased renal metabolism ↑ blood glucose BODY SYSTEMGastrointestinal COMPLICATIONS ∙ Anorexia ∙ Nausea/vomiting ∙ GI bleeding ∙ Pancreatitis ∙ Bowel problems EFFECTS ∙ Accumulation of uremic toxins affects all segments of GI tract ∙ Fluid/electrolyte imbalance BODY SYSTEM Genitourinary COMPLICATIONS ∙ Amenorrhea ∙ Impotence EFFECTS∙ Uremic toxins BODY SYSTEM Hematology COMPLICATIONS ∙ Anemia ∙ Bleeding tendency EFFECTS ∙ Inadequate erythropoietin; platelets have ↓ adhesiveness ∙ Decreased life span of red blood cells (RBCs) BODY SYSTEM Immune COMPLICATIONS ∙ Infections EFFECTS ∙ Altered immune response BODY SYSTEM Integumentary COMPLICATIONS ∙ Dry skin ∙ Pruritus ∙ Ecchymosis EFFECTS ∙ Dry skin ∙ Pruritus ∙ Ecchymosis ∙ Decreased activity of oil glands and decreased size of sweat glands ∙ Phosphate and uremic crystal deposits in skin. ∙ Altered platelet count BODY SYSTEM Metabolic COMPLICATIONS ∙ Metabolic acidosis ∙ Hyperkalemia ∙ Hyperphosphatemia ∙ Hypocalcemia EFFECTS ∙ Retention of metabolic acids; kidney unable to excrete daily H+ ions or K+; increased metabolic acidosis due to phosphate acids BODY SYSTEMMusculoskeletal COMPLICATIONS ∙ Muscle weakness ∙ Bone pain ∙ Bone tenderness ∙ Osteoporosis EFFECTS ∙ Electrolyte imbalance ∙ Hypocalcemia (bones release Ca++ to increase plasma levels of Ca++) BODY SYSTEM Neurologic COMPLICATIONS ∙ Altered thought processes ∙ Headaches ∙ Neuropathy EFFECTS ∙ Uremic toxins and electrolyte imbalance ∙ Decreased nerve conduction

An older male patient visits his primary care provider because of burning on urination and production of urine that he describes as "foul smelling." The health care provider should assess the patient for what factor that may put him at risk for a urinary tract infection (UTI)?

Benign prostatic hyperplasia (BPH) BPH causes urinary stasis, which is a predisposing factor for UTIs. A sedentary lifestyle and recent antibiotic use are unlikely to contribute to UTIs, whereas a diet high in purines is associated with renal calculi.

You teach patients to take Feosol (ferrous sulfate):

Between meals

A patient with a history of end-stage renal disease (ESRD) resulting from diabetes mellitus has presented to the outpatient dialysis unit for his scheduled hemodialysis. Which assessment should you prioritize before, during, and after his treatment?

Blood pressure and fluid balance Although all of the assessments are relevant to the care of a patient receiving hemodialysis, the nature of the procedure indicates a particular need to monitor blood pressure and fluid balance.

Which of the following blood values are monitored when evaluating kidney function? renal colic and nephrolithiasis who has lithotripsy

Blood urea nitrogen (BUN) and creatinine.

You are caring for a patient with a diagnosis of iron-deficiency anemia. Which clinical manifestations are you most likely to observe when assessing this patient?

Brittle nails; smooth, shiny tongue; and cheilosis Brittle nails; smooth, shiny tongue; and cheilosis Specific clinical manifestations may be related to iron-deficiency anemia. Pallor is the most common finding, and glossitis (inflammation of the tongue) is the second most common; another finding is cheilitis (inflammation of the lips). The patient may report headache, paresthesias, and a burning sensation of the tongue, all of which are caused by lack of iron in the tissues.

CAUTI= Catheter associated UTI

Catheter associated UTI -The longer in place, the greater the risk of UTI -Never put in for nursing convenience bacteria enter the bladder by migrating through urine in cath or up urethra outside the cath Bacteria enter thru lower tract; E.coli is responsible for most. Kidney becomes inflamed and edematous; Usually an abrupt onset; Chronic conditions leads to scarring **Older adults may be asymptomatic or may present with nucturia, incontinence, confusion, behavioral changes, lethargy and just not feeling right*

The nurse is assessing a patient with chronic iron deficiency anemia. What finding should the nurse expect during this assessment?

Cheilosis

Urinary How would a UTI manifest in an elderly person?

Chills, coughing, fever, confusion or delirium mental status changes. urine retention, elevated WBC, cloudy foul smelling urine

Explain the relationship between hypertension and chronic kidney disease?

Chronic Kidney Disease (CKD) is a slow, gradual process of kidney destruction. It may go unrecognized for year as nephrons are destroyed and functional kidney tissue is lost. Eventually, the kidneys are unable to excrete metabolic wastes and regulate fluid and electrolyte balance. At this point, the patient is said to have end-stage renal disease (ESRD), the final stage of CKD. CKD may not be identified until uremia develops. Many side effects follow with this, Hypertension is Common and maintaining blood pressure is vital to preserve the kidney perfusion. Burke p. 768 Kidney disease disrupts regulation of the renin-angiotensin-aldosterone system and can lead to salt and water retention causing hypertension. Coarctation (narrowing) of the aorta also occurs, the aortic narrowing reduces blood flow to the kidneys and peripheral vascular system, triggering responses that raise the blood pressure. The kidneys filter out fluid and electrolytes from the blood including sodium and water, to make urine. If the kidneys are not filtering the blood properly, water and electrolytes accumulate and cause fluid overload which manifests by hypertension and edema (Burke 767). On the flip side, hypertension can cause intrinsic acute kidney injury because it causes arteries around the kidney to harden or weaken. These damaged arteries prevent oxygen and nutrients from getting to the kidney, causing damage to the kidney itself . Also, since the kidney creates aldosterone which is a key hormone in the renin - angiotensin - aldosterone system, a system which helps regulate blood pressure, when the kidney is damaged, blood pressures become uncontrolled.

Urinary Incontinence Collaborative Care:

Collaborative Care: • Identify and correct the cause, if possible: Give a diary to track • Physical assessments - check for cystocele (prolapsed bladder) or urethrocele (urethra sags down to vaginal area) or enlarged prostate Check: o PVR (post void residual): 50mL or less is within normal limits o Uroflowmetry (flow and force during urination, used to diagnose pain during urination or urinary incontinence) o Cystometrogram (measures how much the bladder can hold and how much pressure built up inside of the bladder as it stores the urine and how full the urine actually is when person feels that urge to void)- Tests both nerve and muscle function Drugs: Prescribed based on type of incontinence o Stress Incontinence: Duloxetine o Urge incontinence: Inhibit muscle contractions and increase bladder capacity: These drugs have *anticholinergic effects: dry mouth and eyes, constipation, confusion and urinary retention o Estrogen Therapy may be effective in postmenopausal women Surgery: o Prostatectomy o Bladder neck suspension: adds support to the bladder neck and urethra, reducing risk of stress incontinence. One to ligaments and one to the bone: Burch Procedure or MMK Operation • Monitor vaginal drainage or dressing • Monitor color and clarity of urine: *should be pink urine, gradually clear • Encourage activity and ambulation

What are some changes the elderly go through in terms of their kidney function?

Common changes in kidney function for the elderly include: -Nephrons are lost with aging, reducing GFR and kidney mass; changes in aldosterone levels and response to ADH -Kidneys less able to concentrate urine -Decreased bladder muscle strength and capacity; increased difficulty emptying bladder -increased risk of fluid, electrolyte and acid-base imbalances -increased risk of kidney failure -increased risk of drug toxicity

Laboratory Tests: Hematologic Disorders

Coomb's test Hemoglobin electrophoresis D-dimer Serum Iron Studies Schilling's test Bone Marrow Aspiration/Biopsy

A pediatric nurse is giving instructions to a 12-year-old client with sickle cell disease and his parents. To prevent a crisis, the nurse explains that the child should do which of the following? (Select all of the appropriate instructions.) A. Stay away from sick children B. Increase fluids C. Restrict fluids D. Avoid exercise E. Be aware that high altitudes may lead to a crisis

Correct answer: Children with sickle cell disease are more prone to infection so should avoid other children who are obviously sick. The crisis from sickle cell disease results when the sickle-shaped cells become "stuck" in small vessels. Therefore, it is critical for the child to stay hydrated. Although exercise is good for these children, they should be advised to drink plenty of water when they are active. High altitudes may also lead to a crisis as the oxygen content of the cells decreases. Incorrect Answers:C. The client who is at risk of a sickle cell crisis should increase his intake of fluids to promote adequate circulation; he should not restrict fluid intake.D. The client at risk of sickle cell crisis should engage in regular physical activity to promote circulation and improve strength. The client should drink plenty of fluids, but should not avoid exercise.

Urinary Tract Infections Medications:

Cystitis: 3- 7- 10 days ABT- can take up to 3 weeks to get rid of organism Pyelonephritis: 10- 21 days ABT Sulfonamides: trimethoprim o Be sure to check allergies 1st o Use sunscreen to prevent sunburn, should avoid sun Urinary anti-infectives: nitrofurantoin -Increase fluid intake of 1,500-2,000 mL/day *Give with meals *Use sunscreen to prevent sunburn, should avoid sun Urinary analgesic: phenazopyridine o For pain, burning, frequency and urgency associated with UTIS. Does not treat infection and must be used along with antibiotic therapy o May turn urine a reddish orange color- may stain clothing *Do not use medications for more than 24-48 hours*

Urinary Tract Infections Cystitis:

Cystitis: inflammation of the bladder Dysuria-difficult or painful urination Frequency Urgency-a sudden, compelling need to urinate Nocturia-voiding two or more times at night Pyuria-presence of pus in the urine (cloudy appearance, foul odor) Hematuria-blood in the urine Lower abdominal (suprapubic) discomfort

The nurse is providing care for a patient who has been admitted to the hospital for the treatment of nephrotic syndrome. What are priority nursing assessments in the care of this patient?

Daily weights and measurement of the patient's abdominal girth Peripheral edema is characteristic of nephrotic syndrome, and a key nursing responsibility in the care of patients with the disease is close monitoring of abdominal girth, weights, and extremity size. Pain, level of consciousness, and orthostatic blood pressure are less important in the care of patients with nephrotic syndrome. Abnormal calcium and phosphorus levels are not commonly associated with the diagnosis of nephrotic syndrome.

Age Related Changes in Kidney Function

Decreased GFR; By age 80, GFR less than 50% of rate at age 30 Decreased # of functional nephrons, change in aldosterone level and response to ADH Kidneys less able to concentrate urine-> nocturia May be decreased K+ excretion Increased risk of fluid, electrolyte and acid-base imbalances. Increase fluid intake. Decreased thirst mechanism increases risk for dehydration. Increased risk of kidney failure. Monitor UO and renal function labs. Increased risk of drug toxicity Avoid nephrotoxic drugs.

Overflow UI

Definition:Loss of urine associated with a overfilling of bladder (urine retention). Usually void frequently in small amounts Cause: Outlet obstruction or impaired detrusor muscle activity Spinal cord injury Neurologic disorder or trauma Enlarged prostate Fecal impaction Anticholinergics Symptoms:Post-void residuals present Cystocele or urethrocele Interventions MEDS Administer medications, as indicated. Anticholinergics (e.g., oxybutynin [Ditropan]) or β-3 agonists (e.g., mirabegron [Myrbetriq]) are often used to reduce bladder spasms that are impeding urine outflow. For some men with an enlarged prostate, treatment with an alpha-adrenergic blocker (e.g., doxazosin Proscar], tamulosin [Flomax]) can help relax the muscle at the base of the urethra and allow urine to pass from the bladder. SURGERY

Urine Retention

DefinitionNormal bladder emptying is prevented by obstruction to urine flow or by a functional problem Cause: BPH (enlarged prostate) Abdominal or pelvic surgery Medications Neurologic diseases Trauma Nurses' bladder Postpartum Symptoms: Firm, distended bladder; may be displaced to one side of midline Inability to empty bladder completely Interventions

Describe the Z-track method in administering medications. Which common meds are given via z-track method?

Describe the Z-track method in administering medications. Which common meds are given via z-track method? Z-Track Technique Before the Procedure Verify medication order, including the patient, drug, dose, route, and time. Gather all supplies. After drawing the correct dosage into the syringe, draw an additional 0.2 mL of air into the syringe and change the needle. Identify the patient and provide for privacy. Position appropriately for the site to be used (prone or side-lying). Procedure Follow Standard Precautions. Identify and cleanse the site. Using the nondominant hand, pull the skin laterally approximately 1-11⁄2 in. (2.5-3 cm). Holding the skin taut, administer the medication and withdraw the needle. Release the skin and apply direct pressure to the injection site. Do not massage. Document the medication administration, site, and patient response..(Pg 484 Burke) Which common meds are given via z-track method? Common drugs given z-track vistaril, and iron

Urge Incontinence

Description: Inability to stop urine flow long enough to reach toilet Pathophysiology: overactive detrusor muscle→ inability to inhibit voiding Causes: Decreased bladder capacity Bladder irritation CNS disorders Treatment: Anticholinergic drugs

BENIGN PROSTATIC HYPERPLASIA Uroflowmetry:

Determine the degree of urethral obstruction

Blood Loss Anemia Diagnostic Tests: Blood Transfusion: Rules: Two nurses must check the label on the: Nursing Implications:

Diagnostic Tests: o RBCS, hemoglobin and hematocrit levels would be low Blood Transfusion: o When blood is coming out, way to fix it is putting it back in. o Done to replace blood loss during surgery or due to serious injury; an also be done when body can't make blood properly because of an illness. o Involves the administration of blood component, specifically whole blood o 1st Check agency procedure, how often to check VS Rules:• o Must give informed consent • o Then Type and Cross-Match: looking for compatibility of the blood, ABO types and Rh groups -o Donor blood is generally screened for infectious diseases before it gets to the hospital -o Some people/family can donate their own blood if they know about their upcoming surgery o Peripheral IV is established -o Remember blood is thick- so use 18 gauge needle -o Use Y-tubing: Blood transfusing with normal saline -o Blood is handled delicately: Don't massage the bag because it breaks down blood cells -o Blood must be star ted within 30 minutes of arrival and should not last longer than 4 hours** • Increasing risk for contamination and infection rates Two nurses must check the label on the: Very Important** o 1. Blood product bag o 2. Blood administration form of the blood bank o 3. Patient's armband and blood bracelet - ****Check policy to make sure this can be delegated to a LPN Nursing Implications o :Reactions still may occur with Transfusions o Take and record baseline vital signs o Stay with the patient during the first 15 minutes of the transfusion- watch for reaction* o Monitor for redness, swelling, SOB, itching, restlessness, fever o *Tell patient to report any warm feelings, chills, itching, feelings of weakness or fainting, or difficulty breathing o Usually take VS every 30-60 minutes until transfusion is completed o Some patients may be pre-medicated: Tylenol (keep fever done), Lasix's, Benadryl o An older patient, 84 years may need transfusion to be nice and slow up to 4 hours* Don't rush it If transfusion reaction occurs, take the following actions: o **Stop the transfusion immediately: alert charge nurse o Disconnect blood tubing and start normal saline slowly- to maintain IV line o *Then send the blood bag to the blood bank laboratory* o Collect urine and venous blood samples

Anemia: Interdisciplinary Care

Diagnostic Tests: Complete blood count (CBC) Iron levels and total iron-binding capacity Serum ferritin Sickle-cell screening test Hemoglobin electrophoresis Schilling's test Bone marrow examination test Medications: Iron Sources Vitamin B12 Sources Folic Acid Sources Know purpose, nursing implications and client teaching for med Dietary Modifications: Blood Transfusions: Prioritizing Nursing Care, Health Promotion, Assess Potential Complications: Nursing Diagnoses: Activity Intolerance: Impaired Oral Mucous Membrane: Self-care Deficit: Evaluating, Documenting & Continuity of Care

BENIGN PROSTATIC HYPERPLASIA Diagnostic Tests

Digital Rectal Exam: reveals an enlarged and asymmetrical prostate gland. o Insert a lubricated, gloved finger into the rectum to palpate the posterior surface of the prostate gland o Should be firm, but soft. Lumps indicate abnormality

Urinary Retention What is it?

Disruption of normal bladder emptying:

KidneyFailure Patients with fistulas have certain restrictions to keep the fistula safe, what are the restrictions?

Do not take blood pressure or allow blood draw or IV starts on the non dominant arm of patients in renal failure, to avoid damaging the fistula or graft, avoid taking blood pressures or doing veni punctures on the affected arm.

Which of the following factors should you include in your discharge teaching plan for patient renal colic and nephrolithiasis who has lithotripsy.

Drink lots of fluids daily, keep active and monitor urine concentrations.

Urinary Medications

Drug Classification: Fluroquinolones fluoroquinolones Drug/Usual Route/Dose: Ciprofloxacin Levaquin Ofloxacin Expected Action:anti-infectives, Death of susceptible bacteria. Nursing Implications/ Patient Teaching: Avoid use in pregnancy Food, calcium, antacids interfere with absorption Tendon rupture Drug ClassificationSulfonamides Drug/Usual Route/Dose: Trimethoprim/sulfamethoxazole (Bactrim) Expected Action:Bactericidal action against susceptible bacteria.Combination inhibits the metabolism of folic acid in bacteria at two different points. Nursing Implications/ Patient Teaching: Provide adequate fluids Sulfa allergies common Drug Classification:Urinary Anti-infective Drug/Usual Route/Dose:Nitrofurantoin (Macrobid) Expected Action Interferes with bacterial enzymes. Nursing Implications/ Patient Teaching: Food or milk reduces GI upset Drug Classification:Urinary Analgesic Drug/Usual Route/Dose:Phenazopyridine (Pyridium) Expected Action: Acts locally on the urinary tract mucosa to produce analgesic or local anesthetic effects. Has no antimicrobial activity. Nursing Implications/ Patient Teaching: Urine may turn orange and stain clothing Give with food Drug Classification: Antispasmodic/ Anticholinergic Drug/Usual Route/Dose:Tolterodine (Detrol) Oxybutynin (Ditropan) Expected ActionTreatment of overactive bladder function that results in urinary frequency, urgency, or urge incontinence. Nursing Implications/ Patient Teaching:Assess safety, constipation, urinary retention Drug Classification: BPH Meds Drug/Usual Route/Dose: Tamulosin (Flomax) Finesteride (Proscar) Cholinergic meds (Urecholine) Expected Action: Inhibits the enzyme 5-alpha-reductase, which is responsible for converting testosterone to its potent metabolite 5-alpha-dihydrotestosterone in prostate, liver, and skin; 5-alpha-dihydrotestosterone is partially responsible for prostatic hyperplasia and hair loss. Therapeutic Effect(s): Reduced prostate size with associated decrease in urinary symptoms. Decreases hair loss; promotes hair regrowth. Nursing Implications/ Patient Teaching: Postural hypotension Impotence

Given patients glossitis, which of the following recommendations is appropriate?

Eat soft foods

To help meet the goal of a normal hemoglobin level, which of the following foods do you encourage patients to include in their diet?

Eggs

Benign Prostatic Hyperplasia What is it?

Enlargement of the prostate gland: Non-cancerous, in the prostate, hyper-growth of cells. o When enlarged, the prostate may obstruct urine flow from the bladder and out the urethra o Family Hx- risk factor

A patient with a DVT is started on enoxaparin and is being sent home with enoxaparin injections twice a day. What teaching would be included?

Enoxaparin may cause bleeding or hemorrhage so the patient should notify their healthcare provider if they have bleeding gums, nosebleeds, unusual bruising, black, tarry stools, blood in the urine, or low blood pressure. This drug may also cause anemia, hyperkalemia, dizziness, itching, rash, fever, swelling, edema, and difficulty breathing. The healthcare provider should be contacted immediately if they experience any of these issues. Aspirin, naproxen, and ibuprofen should not be taken with enoxaparin, unless otherwise instructed by the physician.The patient should be taught the correct technique for injection. Instructions should be given to administer deep into the subcutaneous tissue, alternating between the left and right anterolateral and left and right posterolateral abdominal wall. They should pinch the skin between the thumb and forefinger and hold while injecting at a 45 to 90 degree angle making sure to inject the entire length of the needle. The patient should not massage or aspirate the injection site. The site should be assessed for hematomas, inflammation and ecchymosis (Nursing Central, McCuistion pgs. 581-582)

Although patient has already been told that he needs to use a urinal and strain his urine, this is reinforced with him at this time. patient is reminded that his urine is being strained to: renal colic and nephrolithiasis who has lithotripsy.

Ensure that any passed stones are captured so that their composition can be analyzed.

Nursing Diagnosis:

Excess Fluid Volume: Record I&O, assess HR and Breath sounds, peripheral edema and JVD, place in fowler's position, restrict fluids, administer diuretics, monitor serum electrolytes Imbalanced Nutrition: Less than Body Requirements: Monitor and document food intake, administer antiemetic drugs 30-60minutes before eating, provide mouth care, small, frequent meals, monitor albumin levels Risk for Infection: Use aseptic technique, temp, WBCS, culture urine, encourage coughing and deep breathing, restrict ill family members from visiting Disturbed Body Image: Involve patient decision making encourage self-care, provide positive reinforcement

Glomerulonephritis Nursing Diagnosis:

Excess Fluid Volume: Restrict sodium, weigh daily, I&O Fatigue: Provide rest periods with procedures and activities Risk for infection: When you have decreased albumin and immunoglobulins-lose ability to fight off infection *Make sure patient is updated with Vaccines*

Nephrotic Syndrome The priority NANDA nursing diagnosis for this disease is:

Excess Fluid volume

urinary incontinence Nursing Diagnosis:

Exercises should be continued for life: Decrease stress and urge o Patient should experience fewer episodes of unintended urine loss o Patient will experience no incontinent episodes related to difficulty responding to urge to void • o Patient will engage in social activities with activities *Anyone with incontinence should go see urologist.

Nephrotic SyndromeThe type of diet recommended is, indicate high or low: What type of diet with azotemia?

Fluid: ​low Na: ​low Protein: moderate restriction restricted sodium intake (if hypertensive or have severe edema) and dietary protein restricted

GlomerulonephritisI will be on this type of diet, indicate high or low:

Fluid: ​low if edema or hypertnsive. Na:​ Restricted if edema is significant or the patient is hypertensive. Low if the patient has nephrotic syndrome. Protein:​ High if proteins are being lost in the urine. if azotemia is present dietary proteins are restricted. When proteins are restricted, those included should be complete proteins meat, fish, egg, soy, or poultry.

Folic Acid Deficiency Anemia

Folic Acid is required for production & maturation of RBC's More common in chronically malnourished Signs/symptoms: similar to Vitamin B12 deficiency anemia except for ___________ Diagnostic Tests: Interdisciplinary Care: Medications Dietary Modifications

Individuals with folic acid deficiency also can be malnourished. What labs indicate malnourishment?

Folic acid deficiency anemia is more common among people who are chronically malnourished. Folic acid is a required part of RBC formation and maturation. When one of these parts is missing, it can lead to anemia. Labs that could incicate folic acid deficiency include a decreased serum folate, decreased Red blood cell count and decreased hematocrit and hemoglobin. Malnurishment can also be indicated in lab results. Low prealbumin, transferrin, creatinine, and BUN levels can all be seen in malnurished patients. (Medscape 2018)(Burke p. 479-480)

When caring for a patient with a history of alcoholism, the nurse recognizes that the patient is at increased risk for which hematologic problem?

Folic acid deficiency=Folic acid deficiency is commonly seen in people who are malnourished, such as in alcoholics. These people suffer from fragile, big and immature RBCs.

When caring for a patient with chronic kidney disease (CKD), the nurse should plan to administer which medication?

Folic acid supplement Patients with CKD have issues with erythropoietin. They will be anemic and need supplementation with folic acid to boost their Hgb cells.

Explain the teaching involved with someone with a fistula in their arm.

For long time access for hemodialysis, an arteriovenous fistula is commonly created, usually on the non dominant arm. Often the radial artery and cephalic vein are joined. A functional fistula has a palpable pulsation and bruit (audible murmur) on auscultation. It is important to note that blood pressure or blood draws should not be done on the affected arm and the reason is to avoid damaging blood vessels. -watch for signs of infection -watch for signs of blood clots -do not take blood pressures or veni punctures on the arm with the fistula. -check fistula daily to check for blood flow by feeling for a thrill or a hearing for a bruit. -do not lift heavy items with fistula arm -check for swelling, redness, bleeding and pain -keep area clean -beware of psychological impact

Urinary Calculi (Urolithiasis): Kidney Stones Uric Acid Stones:

Form from people who don't drink enough fluids or lose too much fluid and patients who have gout

Urinary CalculiDietary recommendations to prevent stone formation:

F​luid intake increased 2.5-3 L per day to prevent concentration of stone-forming salts. (Take 1 or 2 drinks at night) low-sodium restricted protein diet (more effective against calcium stones than reducing calcium) Oxalate (foods found in spinach nuts & chocolate s/b limited in pts with calcium stones) Organ meats, sardines and other high-purine foods eliminated from diet who forms stones with Uric acid.

To prevent injury from dizziness when getting out of bed in the morning, you teach

Get out of bed slowly

To prevent injury from dizziness when getting out of bed in the morning, you teach patients to

Get out of bed slowly

Pernicious Anemia (Vitamin B12) Sources of Vitamin B12

Giving Medications Safely: Vitamin B12: (Cyanocobalamin)-**Medication comes in red vial and medication is bright red colored** o Not a multi-use vial - o If severe given every day and then once/month • ----o *Administer injections IM or deep subq. to decrease irritation (deltoid common) ----o *Let patient know that there may be a burning sensation with injection • ----o *Avoid alcohol, which interferes with absorption • ----o *Do not expose injection vial to light Folic-Acid: Helps produce DNA: Risk factors: o -Drinking alcohol: Refrain from drinking alcohol o -Malnourished: older adults o -Pregnant women whose folic acid requirements are increased o -Diseases in which folic acid is not well absorbed in the digestive system in Crohn's or Celiac's disease** Manifestations include: o -Glositis- inflamed tongue o -Cheilosis o -Diarrhea Two serious birth defects may occur: linked to folic acid deficiency during pregnancy o 1. Spina bifida o 2. Anencephaly Foods to eat: o Asparagus, okra, spinach and other greens, broccoli o Green beans and other legumes, dried beans o Fortified grain products (corn flakes, oatmeal, pasta)

Acute Glomerulonephritis

Group A beta-hemolytic Streptococcus (strep throat) Inflammation & damage in glomerular membrane→ capillary walls porous Plasma proteins & blood cells escape into urine Disease of childhood Manifestations: Gross hematuria→ smoky, coffee or cola-colored urine Edema: periorbital and dependent edema HTN, fatigue, anorexia, NV, headache Increased BUN & creatinine

Does having the sickle trait mean you have the disease? If yes, how? If no, why not?

Having sickle cell trait is not the same as having sickle cell disease. Sickle cell disorders are genetically transmitted, inherited as an autosomal recessive trait. In disorders with autosomal recessive inheritance pattern, a person must inherit the abnormal gene from both parents rot the disorder to be fully expressed. Sickle cell disorders are characterized by the presence of abnormal hemoglobin, hemoglobins S, in the RBCs. This abnormal hemoglobin affects how RBCs respond to stress. When blood oxygen levels fall, the cells deform and become sickle shaped. These deformed RBCs clump together and obstruct small blood vessels . Although the shape returns to normal when normal oxygen levels are restored, repeated episodes of sickling damage RBC membranes. The damaged RBC break down (hemolysis) leading to anemia. Events likely to trigger a sickling event include hypoxia, low temperatures, excessive exercise, anesthesia, dehydration, infection, or acidosis. Sickling is usually marked by an abrupt onset of intense pain, often in the abdomen. The acute pain also may occur in the chest, back, or joints. The patient also has manifestations of anemia and may become jaundiced as bilirubin is released when red blood cells are destroyed. Sickle cell disease is when you have abnormal hemoglobin (hemoglobin S) is formed within red blood cells. Sickle cell disorders usually affect people of African descent. About 8% of African Americans have inherited the Hemoglobin S gene from a parent. They have sickle cell trait and have few symptoms. Each child of a person with sickle cell trait has a 50% risk of inheriting the Hemoglobin S gene. If both parents have sickle cell trait, each child has a 25% risk of inheriting the Hemoglobin S gene from both parents. These children are likely to develop sickle cell disease. Hispanics from the Caribbean and Central and South America and people from the Eastern Mediterranean, Arab regions, and India also may have the hemoglobin S gene. Sickled cells obstruct blood vessels, causing tissue ischemia. Every organ of the body can be damaged, especially the spleen, bone marrow, lung eye, and head of the femur and humerus. Other Acute events may develop in patients with sickle cell disease: sequestration crisis (pooling of blood in the liver or spleen); temporary loss of bone marrow function, leading to aplastic anemia; or hemolytic crisis, a rapid drop in hemoglobin levels. Good general health maintenance through nutrition, exercise, and avoiding smoking and excess alcohol intake are important to prevent sickling crisis. Sickle cell disease is chronic, unpredictable, and recurrent, leading to psychosocial stress. Patients with sickle cell disease experience frequent and painful sickling episodes, often leading to hospitalization. The risk of passing the disease to offspring compounds the stress. Genetic counseling is recommended. Both parents have to have the sickle cell trait.

What is the term for blood in the urine: What is the term for foul smelling urine:

Hematuria Pyuria

Which assessment finding is commonly found in the oliguric phase of acute kidney injury (AKI)?

Hyperkalemia In AKI, the serum potassium levels increase because the normal ability of the kidneys to excrete potassium is impaired. Sodium levels are typically normal or diminished, whereas fluid volume is normally increased due to decreased urine output. Thrombocytopenia is not a consequence of AKI, although altered platelet function may occur in AKI.

If a patient is in the diuretic phase of AKI, you must monitor for which serum electrolyte imbalances?

Hypokalemia and hyponatremia In the diuretic phase of AKI, the kidneys have recovered their ability to excrete wastes but not to concentrate the urine. Hypovolemia and hypotension can result from massive fluid losses. Because of the large losses of fluid and electrolytes, the patient must be monitored for hyponatremia, hypokalemia, and dehydration.

patient has had the intravenous pyelogram (IVP) procedure. Post-procedure care involves assessing patient for signs and symptoms of allergic reaction to the dye used for the procedure. These signs and symptoms can include: renal colic and nephrolithiasis who has lithotripsy.

Hypotension, tachycardia, itching respiratory distress

PATIENT TEACHING Pelvic Floor (Kegel) Exercises

I Identify the pelvic muscles by: o Attempting to stop the flow urine during voiding and holding for a few seconds. o Tightening the muscles of the vagina around a gloved finger or tampon. o Tightening the muscles around the anus as though trying to avoid passing flatus. Perform exercises: Tighten pelvic muscles, hold for 10 seconds, and relax for 10 to 15 seconds. Continue the sequence (tighten, hold, relax) for 10 repetitions. o Keep abdominal muscles and breathing relaxed while performing exercises. o Initially, exercises should be performed twice per day, working up to four times a day. o Exercise at a specific time each day or in conjunction with another daily activity (e.g., bathing or watching the news). Establish a routine, because these exercises should be con- tinued for life.

Compare IVP to a cystoscope

IVP issues contrast medium and xray to evaluate the urinary tract. the contrast medium is injected intravenously. It is filtered from the blood by the kidneys, allowing x-rays to show the contrast entering the kidney pelvis, flowing through the ureters, and into the bladder. This allows evaluation of renal function and the position, size, shape, and structure of the urinary tract organs. (measuring the time required for filtration. Cystoscopy is direct visualization of the urethra and bladder using an endoscope. its used to diagnose conditions such as urethral structures bladder stones, tumors, congenital abnormalities. its invasive and performed in an endoscopy laboratory or cystoscopy room of the surgical site.

Pediatric Urinary Tract Differences

In infants, renal function is normal except under stress At 6-12 months old kidney function is near adults Premature infants: reabsorption of glucose, Na+, bicarbonate and phosphate is reduced Young infants cannot concentrate urine as efficiently as older kids & adults d\t immaturity of nephrons Urethras shorter in younger children - risk for UTI Bladder control usually achieved by age 4-5 Most children with ARF fully recover, unlike adults

Glomerulonephritis What is it?

Inflammatory condition that primarily affects the glomerulus; may be an acute or chronic disorder. --Involves the kidneys

A patient taking warfarin (Coumadin) needs to have regular coaguaulation studies done. Which of the following laboratory tests needs to be evaluated? Select all that apply.

International Normalized Ratio (INR) Prothrombin Time (PT)

Urinary Incontinence What is it?

Involuntary urination is a common problem that can cause physical problems; skin breakdown, infection, and rashes, as well as embarrassment, isolation and withdrawal, feelings of worthlessness, helplessness and depression. -Can be due to a lack of pelvic floor muscle tone -Loss of bladder control Urinary continence requires a bladder that is able to expand and contract and sphincters that can maintain a higher pressure in the urethra than in the bladder.

Nutritional Anemias

Iron deficiency Anemia Vitamin B12 (Pernicious) Anemia Folic Acid Deficiency Anemia

Explain the priority nursing management in treating someone with glomerulonephritis?

It is necessary to determine the root cause of glomerulonephritis to treat the disorder. Acute glomerulonephritis is often caused by a group A beta-hemolytic Streptococcus infection, so throat and skin cultures may be done to determine whether group A beta-hemolytic streptococci antibodies are present or not. Penicillin or other antibiotics can be used to treat existing streptococci bacteria. Labs such as serum creatinine, blood urea nitrogen (BUN), and urine creatine clearance levels are important to monitor in the patient with glomerulonephritis in order to assess kidney function. A 24-hour urine specimen may be collected establish the amount of protein and creatinine are in the urine. The patient should also be assessed for fluid electrolyte and acid-base imbalances. Daily weight, amount of edema, and fluid intake and output should be monitored and documented. Lungs should be auscultated to listen for adventitious lung sounds related to fluid retention and edema. Hypertension is commo patients with glomerulonephritis, so vital signs, especially blood pressure and heart rate should be monitored. If the patient is hypertensive, antihypertensive medications may be ordered. The patient should avoid engaging in activities until symptoms subside. A low sodium diet with limited fluids should be followed to treat fluid retention. To replenish protein lost in the urine, the patient should increase protein rich foods such as meat, fish eggs, soy or poultry. Glomerulonephritis is an inflammatory condition that primarily affects the glomerulus. It can be acute or a chronic disorder. It can also be either a primary kidney disorder or secondary to a systemic disease such as lupus erythematosus or diabetes. Glmerulonephritis affects both the structure and function of the glomerulus. It damages the capillary membrane allowing blood cells and proteins to escape into the filtrate. Acute glomerulonephritis usually follows an infection of group A beta-hemolytic streptococcus (ex. strep throat) manifestations usually occur 10-14 days after initial infection. In chronic glomerulonephritis destruction of the glomeruli is slow and progressive, the kidneys decrease in size and their surfaces become granular or rough. This is not usually recognized until signs of renal failure is evident. Manifestations: hematuria (cola or coffee colored), proteinuria, edema (periorbital, facial, dependent), hypertension, fatigue, N&V, anorexia, headache, elevated BUN. Diagnosis: throat or skin cultures, BUN and serum creatinine elevated. Serum electrolytes, urinalysis shows blood cells and protein urine. renal ultrasound may be done to evaluate kidney size. Nursing: there is no specific drug treatment for glomerulonephritis. --penicillin or antibiotics may be ordered with current or history of strep. to kill bacteria. --antihypertensives- to maintain normal blood pressures --ACE inhibitors- or ARB's- protect the kidneys --Glucocorticoids or immuosuppressive drugs- reduce risk of kidney failure --low-sodium diet- reduce edema --increase protein if proteins being lost in urine --(low-sodium/low-protein diet with nephrotic syndrome) --provide comfort and maintain ability to complete ADL's

How would you teach someone to perform Kegal exercises? How often?

Kegal's are done to reduce the incidence of UTI, teach women to perform pelvic floor muscle to improve perineal muscle tone. It may also benefit men who experience UTI after prostatectomy. Kegel exercises improve perineal muscle tone. Identify the pelvic muscles by: --Attempting to stop the flow of urine during voiding and holding for a few seconds. --Tightening the muscles of the vagina around a gloved finger or tampon. --Tightening the muscles around the anus as though trying to avoid passing flatus. Perform exercises: Tighten pelvic muscles, hold for 10 seconds, and relax for 10 to 15 seconds. Continue the sequence (tighten, hold, relax) for ten repetitions.

Eight months after the delivery of her first child, a 31-year-old woman has sought care because of occasional incontinence that she experiences when sneezing or laughing. Which measure should the nurse first recommend in an attempt to resolve the woman's incontinence?

Kegel exercises Patients who experience stress incontinence frequently benefit from Kegel exercises (pelvic floor muscle exercises). The use of incontinence pads does not resolve the problem, and intermittent self-catheterization would be a premature recommendation. Dietary changes are not likely to influence the patient's urinary continence.

Kidney Pelvis Renal colic: Bladder

Kidney Pelvis o May be asymptomatic o Dull, aching flank pain Ureter o Renal colic: Acute, severe, intermittent flank pain on affected side o Pain radiating to suprapubic region, groin, and scrotum or labia o Nausea, vomiting o Pallor; cool, clammy skin o Microscopic hematuria Bladder o May be asymptomatic o Dull suprapubic pain o Microscopic or gross hematuria Hydronephrosis may occur*urine backs up into kidneys

diet

Kidney Stones: Diet: Fluid intake Drink 1-2 glasses of water at night Limit foods Calcium stones

Which finding allows you to identify the patient's anemia as folic acid deficiency rather than cobalamin deficiency?

Lack of neuromuscular symptoms Lack of neuromuscular symptoms The absence of neurologic problems is an important diagnostic finding and differentiates folic acid deficiency from cobalamin deficiency.

How should you assess the patency of a newly placed arteriovenous graft for dialysis?

Listen with a stethoscope over the graft for presence of a bruit. A thrill can be felt by palpating the area of anastomosis of the arteriovenous graft, and a bruit can be heard with a stethoscope. The bruit and thrill are created by arterial blood rushing into the vein.

More KidneyFailure What the nurse expects to see post hemodialysis:

Low electrolyte levels, water and waste products are filtered out. Hypotension can result. and a decrease in weight compared to baseline

Urinary What to watch for post TURP?

Maintain continuous bladder irrigation as ordered, ensure catheter patency. monitor for excess bleeding. encourage fluids 2500-3000ml/day give stool softeners to prevent straining Clots and bright red blood normal is light red urine with small clots

Treatment of Urinary Incontinence

Medications: Urge: Anticholinergic drugs→ inhibit detrusor muscle contractions. Tolterodine (Detrol) and oxybutynin (Ditropan). SE include: dry mouth & eyes, constipation, confusion, urinary retention Stress: If estrogen deficiency, estrogen therapy Surgery: Cystocele, Urethrocele= bladder neck suspension Prostate= prostatectomy Nursing Diagnoses Impaired urinary elimination: Toileting Self Care Deficit: (Functional Incontinence) Social Isolation: Continuing Care: Assess home environment: Voiding diary Restrict fluids in evening Limit bladder irritants Teach Kegel's exercises Discuss HRT, surgery and PT Encourage weight loss

Why are diabetics at increased risk for UTI's?

Microvascular complications involve alterations in the microcirculation (small blood vessels and capillaries) especially the eyes, kidneys and nerves. Diabetic neuropathy involve disorders of the peripheral nerves and the ANS which causes impaired genitourinary function. Neuropathy result from thickening of capillary membrane and destruction of myelin sheaths, which impair nerve conduction. Neurogenic bladder, inability to empty bladder completely, leading to urinary retention and an increased risk for UTI. Also the increase an glucose levels in urine can lead to bacteria growth. When looking at your urinalysis results, positive leukocytes (WBC) will indicate UTI. Burke PG 928-929

More Kidney Failure Post procedure care to avoid nephrotoxicity after CT with contrast:

Monitor BUN and serum creatinine levels. Avoid potentiating drugs (some antibiotics, contrast dye, NSAIDs) Alkalinize the urine by adding sodium bicarbonate to IV infusion and administer allopurinol (zyloprim) or rasburicase for TLS prevention. encourage fluids to excrete the contrast.

When caring for a child with nephrotic syndrome, the nurse would include which of the following in the plan of care? Select all that apply.

Monitor for increased urine protein Teach parents to watch for manifestations of thromboemboli. Assess for periorbital and dependent edema Monitor for low serum albumin levels.

child with nephrotic syndrome, the nurse would include which of the following in the plan of care?

Monitor for increased urine protein Teach parents to watch for manifestations of thromboemboli. Assess for periorbital and dependent edema Monitor for low serum albumin levels.

Kidney Failure Medications to be avoided are:

NSAIDs, nephrotoxic drugs.

Kidney FailureDiet recommendations for chronic kidney disease, indicate high or low:

Na: Low Sodium Fluid:Low FluidK: low Protein:Low/limit Carbs:Increased

Nephrotic Syndrome Common complication of this condition:

Nephrotic Syndrome thromboemboli

Nephrotic Syndrome vs Glomerulonephritis

Nephrotic Syndrome What is happening? Changes to the glomerulus of the nephron that leads to the leakage of a MASSIVE AMOUNT OF PROTEINS into the urine. Main Causes: Changes to the glomeruli (cause not 100% known) Minimal change disease (most common cause) this disease shows glomeruli changes when assessed with an electron microscope Tends to affect the pediatric population (ages 2-5 years) Other causes can be secondary due to a disease causing changes to the glomeruli, such as lupus, diabetes, heart failure, etc. Various types of proteins can be lost: Main protein lost is ALBUMIN...can also lose immunoglobulins that help fight infection and proteins that decrease clot formation. Signs and Symptoms of Nephrotic Syndrome: Massive Proteinuria (>3 Grams per day) Foamy Frothy urine dark yellow Hypoalbuminemia (low levels of albumin in the blood) Hyperlipidemia: as the liver tries to make more albumin it will produce more triglycerides and cholesterol as wellEdema: weight gain...starts in face and around the eyes and progresses to the extremities and abdomen Main Nursing Concerns: Monitor fluid status and swelling (diuretics and IV albumin may be ordered by the physician to help with swelling and replace low albumin levels) At risk for infection (especially the edematous skin) Corticosteroids or immune suppressors to help decrease amounts of protein lost in urine...prevent relapse per physician's order Assess for blood clots: deep vein thrombosis, respiratory change pulmonary embolism etc. Diet limiting sodium, fluid, and fat Educate about relapse and how to monitor for signs and symptoms Acute Glomerulonephritis What is happening? Inflammation of the glomerulus that leads to the leaking of RED BLOOD CELLS and PROTEIN (mild) into the urine. Main Cause: Post streptococcal infection (occurs 14 days after a strep infection of the throat or skin...impetigo)Tends to affect the pediatric population (ages 2-10) Understanding how it works: AGN is not caused from the strep bacteria attacking the glomerulus but the immune system's response to the bacteria by creating antigen-antibody complexes, which inflames the glomerulus. This is why you see it AFTER a strep infection. Signs and Symptoms of AGN: "Had Strep" Hypertension ASO antistreptolysin titer positive (test used to diagnose strep infections) Decreased GFR: inflammation due to complexes in the glomerulus (low urine output)...oliguria (watch potassium level) Swelling in face/eyes (edema)...mild Tea-colored urine (cola colored)...from hematuria Recent strep infection Elevated BUN and creatinine Proteinuria (mild) Main Nursing Concerns: Monitor blood pressure (antihypertensive medications) and diuretics may be ordered by the physician. Monitor fluid status: intake and output...if oliguric watch for hyperkalemia because when the GFR is decreased potassium is not excreted properly, and swelling Diet limiting sodium and fluid...if oliguria presents will need to limit potassium-rich foods as well. Relapse not common but teach how to monitor for strep infections in the future and the importance of obtaining throat/skin cultures

Explain the priority nursing management in treating someone with nephrotic syndrome?

Nephrotic syndrome results when glomerular tissues are damaged and there is significant protein lost in the urine. It's characterized by proteinuria, low serum albumin levels, high blood lipids, and severe edema that may affect the face and periorbital area. The common complication of nephrotic syndrome is thromboemboli (mobilized blood clots). The main nursing concerns for a pateint with nephrotic syndrome would be: --Monitor fluid status and swelling (diuretics and IV albumin may be ordered by the physician to help with swelling and replace low albumin levels) --At risk for infection (especially the edematous skin) --Corticosteroids or immune suppressors to help decrease amounts of protein lost in urine...prevent relapse per physician's order --Assess for blood clots: deep vein thrombosis, respiratory change pulmonary embolism etc. --Diet limiting sodium, fluid, and fat --Educate about relapse and how to monitor for signs and symptoms Source: BURKE pg 750/751

Urinary Incontinence in Older Adults

Never considered normal Bladder capacity declines Involuntary contractions Decreased estrogen levels Pelvic muscles relax Risk factors: immobility, chronic degenerative diseases, dementia, medications, DM, stroke Increases risk for: falls, fractures, pressure ulcers, UTI's and depression

UrinaryIncontinence Unable to get tothe toilet quickenough becauseof my stroke:

Non- obstructive UrinaryRetention Weak bladder muscle and nerve problems that interfere with signals between the brain and the bladder.Common causes STROKE could also be functional b/c of limited mobility.

Nephrotic Syndrome

Not a disease, but a group of symptoms Result of damage to glomerular membranes with severe protein loss in urine Manifestations: Proteinuria Hypoalbuminemia High blood lipids Edema Thromboemboli Usually resolves in kids Adults may have long-term effects: persistent proteinuria and progressive renal failure

A 24 hour urine collection is ordered for a patient. How should this be done? If the collection gets contaminated or forgotten to be placed in the container, how should the nurse proceed?

Nursing Care Checklist o Note any diet or medication changes required during the collection period. o Obtain specimen container with preservative (if indicated). Label with identifying data, the test, time started, and time of completion. o Obtain a clean urine-collection device and place it in the room. o Post notices - on chart, in Kardex, on the door, over the bed, and over the toilet - alerting all personnel that all urine is to be saved. o At start time, have patient empty bladder completely and discard urine. o Save all urine for next 24 hours in the container, placing container on ice or refrigerating as indicated. If any urine is missed, the collection must be restarted. o At end of collection time, have patient empty the bladder completely and save this urine in the container. Take entire specimen with requisition to the lab. o Chart appropriately. o Teaching: Save all of your urine for the 24 hours, urinate (and save) before moving bowels, do not put any toilet tissue in the urine container. o If any urine during the 24 hours is missed or contaminated, as stated above, all of the previously saved urine must be dumped and the specimen collection must be completely restarted. Remind the patient and other staff of the collection process so an error does not occur again.

Kidney Failure Acute Kidney Injury:

Occurs when the kidney suddenly become unable to filter waste products from blood o Rapidly develop over a few hours or days; common in critically ill patients in acute care. Reversible in prompt treatment --Most common cause is ischemia: poor perfusion and nephrotoxins Prerenal: Blood supply Intrinsic: inflammation Postrenal: Obstruction o Sharp drop in GFR; lose ability to filter: increased BUN or Serum Creatinine levels o Oliguria, urine output less than 400 mL/day, may be present

What medications are given and held on the day of dialysis?

On the day of dialysis medications held include: o Antihypertensives (ACE inhibitors and Beta blockers) - Antihypertensives should be taken after dialysis to prevent hypotension. Blood pressure should be regularly checked during dialysis to make sure it is under control. It may need to be held, as dialysis causes hypotension . o Antibiotics/ antifungals/antivirals- are removed during dialysis, should be held day of, can be taken at bedtime o ASA (Aspirin)- can be removed by dialysis, administer at bedtime o Anticonvulsants (phenytoin, gabapentin)- should be given after dialysis unless urgently needed for seizure o Vitamin B and C (Replavite multivitamin)- these vitamins are best taken after dialysis, as they are often lost during the process. These medications are given on day of dialysis or during dialysis: o Erythropoietin Stimulating agent (ESAs)- are received on dialysis unit for those patients who don't produce enough red blood cells o Iron- is given by IV during dialysis, many patients on dialysis have low iron. o Phosphate binder (calcium carbonate)- given at the beginning of each meal, prevent phosphate from being absorbed as many dialysis patients have a build up of phosphate o Vitamin D (calcitriol)- often deficient in dialysis patientso

When providing teaching for the patient with iron-deficiency anemia who has been prescribed iron supplements, you should include taking the iron with which beverage?

Orange juice Taking iron with vitamin C (ascorbic acid) or orange juice, which contains ascorbic acid, also enhances iron absorption. Milk may interfere with iron absorption. Ginger ale and water do not facilitate iron absorption.

Urinary Incontinence I have an overactive detrusor muscle:

Overactive bladder incontinence: Overactive detrusor muscle increases bladder pressure, causing in ability to inhibit voiding.

Overflow & Reflex Incontinence

Overflow: urinary retention→ overdistention & loss of small amounts of urine Reflex: Loss of moderate amount of urine without warning Pathophysiology: Outlet obstruction or impaired detrusor muscle activity leads to overfilling of bladder & increased pressure (overflow). Altered spinal cord activity→ hyperreflexia of the detrusor muscle (reflex). Causes: Neurological disorders or trauma (overflow & reflex) Enlarged prostate Fecal impaction Anticholinergic drug effects (overflow)

A patient with pneumonia develops kidney stones while hospitalized. While the patient is being evaluated for lithotripsy what should the nurse identify as the priority for this patient's care?

Pain control

Someone with kidney or bladder stones is on pain meds, has to drink plenty of fluids and has to strain their urine. Explain the rationale behind each of these interventions?

Pain meds-Pain relief is vital in treating a patient with kidney stones because they can cause severe pain while making their way down the ureter. Pain medications should be administered on a schedule to keep pain at a 3 or lower on a pain scale of 0 to 10. Fluids- An increase in fluid intake increases urinary output, which helps to move the stone through the ureter and decrease pain. Straining all urine- straining all urine for stones and savings recovered stones for laboratory analysis helps to direct treatment to prevent further stone formation.

Patients with pernicious anemia have paresthesias, why is this?

Paresthesias develop because vitamin B12, which is deficient in pernicious anemia, is important for neurological function. Paresthesias (altered sensation/numbness/tingling in the extremities) and problems with proprioception, (sensing one's position in space/in relation to something), are manifestations of this B12 deficiency. B12 deficiency is typically caused by malabsorption of it in the GI tract related to inadequate intrinsic factor which is necessary for B12 absorption. (Burke, pg 479) remember that food items that contain caffeine are not recommended for people with anemia as it affect the absorption of iron.

White Blood Cells/ Leukocytes

Part of body's defense system again infection and disease Granulocytes: 3 types 1. Agranulocytes 2. Leukocytosis 3. Leukopenia

Why do patients with chronic kidney disease appear confused at times? How does uremia come into play here? Which labs should be monitored?

Patients with CKD will appear confused at times due to the toxins and metabolic wastes that are accumulating in the kidneys and not be excreted out of the body (azotemia) which destroys and targets the brain and nephrons. In the early stage 1, nephrons are being destroyed but the unaffected nephrons will do extra work to replace the lost nephrons so the patient is free of symptoms. In stage 2, patient is still symptom free even with a slight decrease in GFR. In stage 3, GFR is moderately decreased. By stage 4, with the continuous decline of GFR uremia develops and is no longer treatable. Treatment would focus on preserving kidney function and slowing the progress of ESRD (end stage renal disease). Early symptoms of uremia are nausea, apathy, weakness, and fatigue. As it progresses may develop frequent vomiting, increasing weakness, lethargy, and confusion. Neurologic effects of ESRD and uremia: --Fatigue, depression, irritability, impaired thinking, insomnia, restless leg syndrome, and paresthesias Labs to monitor: --serum creatinine, BUN (to evaluate disease progress and its treatment) --GFR (to monitor renal function) --serum electrolytes, ABGs, and CBC --urinalysis (urine albumin to creatinine ratio, blood cells, cell casts, and protein)

What type of diet needs to be taught in someone with chronic kidney disease and why? Explain the pathophysiology.

Patients with chronic kidney disease my be put on a sodium and fluid restriction due to the fact that the kidneys cannot effectively regulate fluid and electrolyte balance and eliminate metabolic waste products. Fluid intake is calculated by adding 500 to 800 mL to the amount of urine output the previous day. Potassium is also restricted. Patients with CKD are also put on a low-protein, high-calorie diet. Protiens that are rich in amino acids are encouraged. restricting protein can reduce uremic symptoms and slow the progression of the disease. Total parenteral nutrition (TPN) may be used if the patient is unable to tolerate food. a fluid restriction is started in order to prevent hypervolemia, edema, and hypertension. The output should be about the same as the input and the output should also include respiration, perspiration, and bowel losses (insensible losses). 500 to 800ml for insensible losses should be calculated along with total output. so if urine output is 325 one day the restriction the next day will be between 825-1125. Due to the decreased GFR the RAA system becomes an issue with salt and water retention. As a result a sodium restriction is also implemented to combat water retention. Weight should be monitored regularly and any gain of 5 pounds over 2 days should be reported to the HCP. Uremia also becomes an issue as the kidney's can't eliminate the nitrogenous waste products that are a result of protein metabolism. Protein intake is then also limited but the proteins that are taken in should be rich in amino acids such as fish, eggs, meat, milk, and soy. Due to this altered diet carbohydrates are increased to maintain an adequate calorie intake.

What is the role of vitamin B12 for patients with pernicious anemia?

Pernicious Anemia is an autoimmune disease. The parietal cells of the stomach lining fail to secrete enough intrinsic factor to ensure intestinal absorption of Vitamin B12, the extrinsic factor. Vitamin B12 is a red crystalline substance extracted from the liver, essential for the formation of red blood cells. Deficiency of vitamin B12 results in pernicious anemia. Vitamin B12 is given orally to patients with nutritional vitamin B12 deficiency and by injection to patients with pernicious anemia. Symptoms for pernicious anemia include weakness, sore tongue, paresthesias of the extremities, and gastrointestinal symptoms such as diarrhea, nausea, vomiting, and pain; in severe anemia, there may be signs of cardiac failure. Vitamin B12 (Cyanocobalamin) Trade Names: Anacobin, Bedoz, Nascobal, Rubion Nursing Implications: Do not expose injection to light, do not mix in the syringe with other medications, administer injections IM or deep subcutaneously to decrease local irritation, monitor hemoglobin,, RBC, reticulocyte counts, and potassium levels. Patient Teaching: Burning sensation that may occur with injection is temporary, avoid alcohol cause it interferes with absorption, if used to treat pernicious anemia, the medication must be taken for life. Nursing Central, Burke p. 482-483 Due to the lack of intrinsic factor, patients who have had gastric bypass surgery are at risk for pernicious anemia.

Platelets/ Thrombocytes

Platelets/ Coagulation studies: Platelets Bleeding time PT time (prothrombin or protime) INR (International Normalized Ratio) PTT APTT

For the patient with AKI, which laboratory result would cause you the greatest concern?

Potassium level of 5.9 mEq/L Hyperkalemia is one of the most serious complications in AKI because it can cause life-threatening cardiac dysrhythmias.

Which of the following foods would the nurse encourage the mother to offer to her child with iron deficiency anemia?

Potato, peas, and chicken Rationale: Potato, peas, chicken, green vegetables, and rice cereal contain significant amounts of iron and therefore would be recommended. Milk and yellow vegetables are not good iron sources. Rice by itself also is not a good source of iron.

Anemia Potential Complications: Risk Factors: Physical Exam:

Potential Complications: o Arrhythmias o Pregnancy Complications- due to low levels of iron and folic acid o Death- such as sickle cell anemia o Blood loss leading to Acute Severe Anemia- can be fatal Risk Factors: o Women of child bearing age are at higher risk due to monthly blood loss from menstrual period o Older adults o Diets low in iron o IBD (Inflammatory bowel disease)- can have up to 30 bloody stools per day leading to anemia Physical Exam: o Inspect skin; look at mucous membranes, conjunctiva and nail beds • o Listen to heart for rapid heart rate (tachycardia) • o Pelvic or rectal exam to look for blood loss • o CBC- tests Hemoglobin (iron riched protein) Hematocrit- measure of how much space RBCS take up in the blood. Tests would show low levels of both in patients with anemia • o MCV (Mean Corpuscular Volume) - a measure of the average size of RBCS in acute cause of anemia *In iron deficiency anemia RBCS are smaller in size then they should be: hypochromatic

What are the pre and post assessment findings in a patient undergoing hemodialysis?

Prior to initiation of dialysis a nurse will assess a patient's lung sounds, check for peripheral edema, pre-weight against estimated dry weight, blood pressure and medications. If a patient is fluid overload the patient will be hypertensive, high pre-weight, have peripheral edema and decreased lung sounds. After dialysis a nurse will auscultate lung sounds for a change in sound, check for decrease in peripheral edema, check post weight and blood pressure for effectiveness of treatment. Before and after treatment the access whether its a fistula or graft should be checked for s/s of infection and the thrill and bruit. Chronic Kidney Disease (CKD) is a slow gradual progress of kidney destruction. At this point, the patient is said to have End-Stage Renal Failure (ESRF). Diabetes is the leading cause of CKD and ESRF, followed by hypertension and glumerulonephritis. In this final stage (ESRF), renal replacement therapies such as kidney transplant or dialysis are necessary. Dialysis is a continuous renal replacement therapy involving the process of cleansing the patient's blood. It involves the diffusion of dissolved particles from one fluid compartment into another across a semipermeable membrane. the dialyzer, one part of extracorporeal system used during hemodialysis treatment, is referred to as the "artificial kedney", its function is to remove excess wastes and fluid from the blood. The patient's blood flows through one fluid compartment of dialyzer filter, the dialysate (bath composed of water and electrolytes), is in another compartment. The both ( blood and dialysate) are inside the same dialyzer but in two different compartments and do not touch each other. Hemodialysis cleans the blood of accumulation of waste products, removes the byproducts of protein metabolism such as urea, creatinine, and uric acid from the blood. It also removes excess body fluid, maintains or restores the buffer system of the body, and corrects electrolytes levels in the body. Treatment varies from one patient to another, most average treatment lasts about four hours and is done three times a week. Before dialysis assessment: Use Standard precaution at all times. Document vital signs, including orthostatic blood pressures (lying and sitting) apical pulse, and respirations and also lung sound and weight. Assess vascular site for palpable pulsation or vibration (thrill), an audible bruit, and signs of inflammation. Promptly report absence of pulsation, thrill or bruit, as well as evidence of inflammation to the charged nurse or physician. Alert all personnel to avoid using the arm with the vascular access site (or nondominant arm if a site has not yet been established) for blood pressure or venipuncture. After Dialysis assessment: Document vital signs, weight, and vascular access site assessment. Monitor orthostatic hypotention, tachycardia, and weight loss. Monitor BUN, serum creatinine, serum electrolytes, and hematocrit. Report possible adverse affects of dialysis such as muscle cramping, headache, nausea and vomiting, altered level of consciousness, seizures, or hypotention. Assess for bleeding at the access site or elsewhere. If transfusion was given during dialysis, report symptoms of transfusion reaction, such as chills and fever; dyspnea; chest, back, or arm pain; and hives or itching. Provide psychologic support; listen actively for feelings of grief, hopelessness, or anger. Refer for social services and counseling as indicated. Teach the patient to monitor his/her puncture site after treatment at home for bleeding, how to apply pressure for 10 min and if not stop go to ER; how to check his/her access every morning for vibration (thrill) and to contact HCP if absent; signs and symptoms of infection (pain, redness, tenderness, fever...), avoid sleeping on his/her access arm, avoid lifting things with the access arm. Bruke p. 771, Saunders p 827.

BENIGN PROSTATIC HYPERPLASIA Medications:

Proscar: aimed to shrink prostate gland: decreases libido and can cause impotence o Medication does not work immediately, takes time to shrink prostate down Proscar: finasteride Hytrin: terazosin (relax the smooth muscle) Flomax: tamsulosin Relax the smooth muscle (relax the smooth muscle) Cardura: doxazosin (relax the smooth muscle) *Helps allow maximum flow

The patient with thrombocytopenia is being started on prednisone. What should the nurse plan as a priority of care?

Protection from infection

A patient informs the nurse that he frequently experiences stress incontinence. Which intervention should the nurse plan for this patient?

Provide patient with absorbent pads or panty liners With stress incontinence, the patient can dribble or leak urine when laughing, coughing...Using a liner will or pad will prevent soiling of clothes.

Urinary Tract Infections Pyelonephritis:

Pyelonephritis: inflammation of the kidney and renal pelvis o Symptoms of cystitis o Flank pain, costovertebral tenderness o Vomiting, diarrhea o Fever, shaking chills o Malaise *The urinary tract is normally sterile above the urethra. How is it kept sterile? • Adequate urine volume • Unimpeded urine flow • Complete bladder emptying • Acidic urine

Laboratory studies are performed for a child suspected of having iron deficiency anemia. The nurse reviews the lab results knowing that which of the following results would indicate this type of anemia?

RBC that are microcytic and hypochromic Rationale: CBC count in children with iron deficiency anemia will show decreased hemoglobin and microcytic and hypchromic red blood cells. The RBC count is decreaed. The reiculocyte count is usually normal or slightly elevated. (Saunders Comprehensive Review for NCLEX-RN 3rd Edition)

Red Blood Cells: Erythrocytes

RBC's & hemoglobin transport oxygen & CO2. The red cells are very numerous. Erythrocytes are without nucleus. Their typical shape is that of a "cake depressed in the center." Biconcave disc:

Blood Loss Anemia What is it?

RBCS and Hemoglobin's are lost from the body with acute or chronic bleeding. Example: Heavy menstrual periods or bleeding in the digestive or urinary tract are considered chronic blood loss. Surgery, trauma and cancers can also cause blood loss. -If there is a lot of blood loss the body may lose enough RBCS to cause anemia. With acute bleeding a patient is at risk for hvpovolemic shock, bleeding may external or internal which is more difficult to assess and chronic bleeding often leads to anemia. -The RBCS are normal in size, shape, and color, but their numbers are reduced.

Laboratory studies are performed for a client suspected to have iron deficiency anemia. The nurse reviews the laboratory results, knowing that which result indicates this type of anemia? A. Elevated hemoglobin level. B. Decreased reticulocyte count. C. Elevated red blood cell count. D. Red blood cells that are microcytic and hypochromic.

Rationale: In iron deficiency anemia, iron stores are depleted, resulting in a decreased supply of iron for the manufacture of hemoglobin in red blood cells. The results of a complete blood cell count in clients with iron deficiency anemia show decreased hemoglobin levels and micocytic and hypochromic red blood cells. The red blood cell count is decreased. The reticulocyte count is usually normal or slightly elevated. (Saunders pg. 561 and 566)

Which nursing intervention takes highest priority when caring for a newly admitted client who's receiving a blood transfusion? 1. warming the blood before the transfusion 2. informing the client that the infusion usually takes 4 to 6 hours 3. documenting the blood administration in the clients chart 4. instructing the client to report any chest pain, dyspnea, or itching

Rationale: This will help the nurse take immediate action in the event of a reaction during the transfusion

The nurse is assisting with the developing a plan of care for a client who is experiencing hematological toxicity as a result of chemotherapy. The nurse should suggest including what in the plan of care? 1. Restricting all visitors 2. Restricting fluid intake 3. Restricting fresh fruits and vegetables in the diet 4. Inserting an indwelling urinary catheter to prevent skin break down

Rationale: in a client who is experiencing hematological toxicity, a low bacteria diet is implemented. This includes avoiding fresh fruits and vegetables, and thorough cooking of all foods. Not all visitors are restricted but the client is protected from those with known infection. Fluids should be encouraged. Invasive measures such as indwelling urinary catheters should be avoided to prevent infections.

A patient is admitted with diagnosis of pernicious anemia. Priority nursing diagnosis would include; a) preventing infection b) increasing iron intake c) providing rest periods d) increasing fluid intake

Rationale; lack of oxygen to the cells result in hypoxia and fatigue. Patients with anemia require frequent rest periods until correction of process can be established. Burke 509, 1331

Prior to administering a client's scheduled dose of enoxaparin sodium (Lovenox), which laboratory finding should the nurse evaluate? 1. PT 2. INR 3. aPTT 4. Platelets

Rationales: A) Prothrombin time (PT) is used when initiating and maintaining therapy with oral anticoagulants, such as warfarin (Coumadin). The PT is a relatively insensitive measure of Lovenox. When thrombolytic studies are indicated the anti-Xa level is used for monitoring clients treated with Lovenox. B) International normalized ratio (INR) is used in conjunction with PT when initiating and maintaining therapy with oral anticoagulants, such as warfarin (Coumadin). The INR is a relatively insensitive measure of Lovenox. When thrombolytic studies are indicated the anti-Xa level is used for monitoring clients treated with Lovenox. C) Activated partial thromboplastin time (aPTT) is measured in clients who are receiving heparin. The aPTT is a relatively insensitive measure of Lovenox. When thrombolytic studies are indicated the anti-Xa level is used for monitoring clients treated with Lovenox. D) The nurse should evaluate the platelet count. This medication is contraindicated in clients with thrombocytopenia. The nurse should also monitor closely for signs of bleeding (hgb/hct and stool for occult blood). The nurse should hold the dose and notify the provider if the platelet count falls below 100,000/mm^3. Enoxaparin sodium is a low-molecular-weight heparin.

Four hours after lithotripsy, patient is able to pass urine without difficulty. He voids 275 mL of bloody urine that contains some gravel. Which nursing actions are indicated? renal colic and nephrolithiasis who has lithotripsy.

Record I/O and send gravel to lab.

Common Lab Tests Hematology:RBC Lab Values

Red blood cell count Reticulocyte count Hemoglobin Hematocrit MCV MCH MCHC

You are evaluating the laboratory data of the patient with suspected aplastic anemia. Which findings support this diagnosis?

Reduced RBCs, reduced white blood cells (WBCs), and reduced platelets Because all marrow elements are affected, hemoglobin, WBC, and platelet values are decreased in aplastic anemia. Other RBC indices usually are normal

When caring for a patient during the oliguric phase of acute kidney injury, what would be an appropriate nursing intervention?

Restrict fluids according to the previous day's fluid loss Patients in the oliguric phase of acute kidney injury have fluid volume excess with potassium and sodium retention. They will need to have dietary sodium, potassium, and fluids restricted. Daily fluid intake is based on the previous 24-hour fluid loss (measured output plus 600 mL for insensible loss). The diet also needs to provide adequate, not low, protein intake to prevent catabolism. The patient should also be weighed daily, not just three times per week.

What predisposes someone to a UTI? How can they be prevented? Burke 745

Risk factors for Uti I Instrumentation of the urinary tract (e.g., catheterization, cystoscopy) 1 Structural abnormalities, obstructions, or strictures I Incomplete bladder emptying I Chronic diseases such as diabetes Females I Women are more likely to experience UTI than men. Short, straight urethra affects 20% of adult women I Proximity of urinary meatus to the vagina and anus I Tissue trauma and possible contamination during sexual intercourse - Use of a diaphragm for birth control I Personal hygiene practices Males I An enlarged prostate gland I Men over the age of 50 Older Adults I The incidence of UTI increases with aging in both sexes. - Unfortunately, hospital acquired infections are among the most common UTIS. I More alkaline (less acid) urine - Higher incidence of diabetes and glucosuria (glucose in the urine promotes bacterial growth) I Incomplete bladder emptying, urinary retention - Changes in vaginal vaginal pH in women; decreased prostatic secretions in me I patients in hospitals and long-term care facilities

Electrolytes lab values

SOdium: 135 - 145 mEq/L Potassium: 3.5 - 5 Calcium: 8.6 - 10.2 md/dL Magnesium: 1.3 - 2.3 mEq/L Ammonia: 15-50 µmol/L Ceruloplasmin: 15-60 mg/dL Chloride: 95-105 mmol/L Copper: 70-150 µg/dL Creatinine: 0.8-1.3 mg/dL Blood urea nitrogen: 8-21 mg/dL Ferritin: 12-300 ng/mL (men), 12-150 ng/mL (women) Glucose: 65-110 mg/dL Inorganic phosphorous: 1-1.5 mmol/L Ionized calcium: 1.03-1.23 mmol/L Magnesium: 1.5-2 mEq/L Phosphate: 0.8-1.5 mmol/L Pyruvate: 300-900 µg/dL Total calcium: 2-2.6 mmol/L Total iron-binding capacity: 45-85 µmol/L Total serum iron: 65-180 µg/dL (men), 30-170 µg/dL (women) Transferrin: 200-350 mg/dL Urea: 1.2-3 mmol/L Uric acid: 0.18-0.48 mmol/L Zinc: 70-100 µmol/L

An electrocardiogram (ECG) is taken. The severely anemic patient may have which of the following ECG changes?

ST-segment depression

The severely anemic patient may have the following ECG changes?

ST-segment depression

Potential Complications: Pyelonephritis:

Sepsis and shock Ex: Urosepsis: urine becomes pus-like, cannot see through it, may be thick in nature -Secondary infection a form of sepsis that initially develops in the urinary tract and then spreads into bloodstream. If infection develops in bloodstream, patients may have a rapid HR and a high fever as body attempts to fight off bacteria. Other symptoms include chills and confusion -Can ultimately cause severe damage to the heart and then to brain because of inadequate oxygen supply -Elderly people can die if UTI is left untreated

What are the significant labs that need to be monitored for someone with chronic kidney disease? Be specific.

Serum Creatinine and BUN are monitored to evaluate the disease progress and its treatment elevation indicates disease getting worse eGFR is monitored to evaluate renal function -- at risk; normal kidney function [early kidney disease may or may not be present >90ml/min. --Mild CKD 60-89ml/min --Moderate 30-59ml/min --Severe CKD 15-29ml/min --ESKD <15ml/min --Creatinine clearance may be ordered Serum electrolytes(hyperkalemia, normal or decreased sodium,hypocalcemia, hyperphosphatemia hypovolemia), arterial blood gas, and CBC Urinalysis may show a fixed specific gravity at 1.010 and abnormal substances such as protein, blood cells, and cell casts. Casts are protein and cellular debris molded in the shape of tubular lumen. Proteinuria is monitored using urine albumin to creatinine ratio. The test requires only a single urine sample and is more accurate than a 24- hour urine protein collection. Kidney biopsy can be done to confirm the underlying disease process.

What are some risk factors for iron deficiency anemia in infants/toddlers?

Several factors can contribute to iron deficiency anemia (IDA) including decrease iron intake, increase iron or blood loss, and periods of increased growth rate. IDA related to inadequate dietary iron intake is rare before age 4 to 6 months because of the presence of maternal iron stores; it occurs most often in children age 9 to 24 months as iron stores are depleted. Premature infants can develop iron deficiency early in life because they are born with insufficient maternal iron stores. Delaying cord clamping by 1 to 3 minutes can decrease the risk for iron deficiency. Decreased iron intake is often related to the intake of large amounts of cow's milk instead of breast milk or fortified formula and iron-fortified foods. (McKinney, pg 1120)

Hemodialysis:

Should feel a thrill and hear a bruit (turbulent sound) -Monitor for bleeding Before Dialysis o Document baseline VS, including orthostatic BP, apical pulse, R, lung sounds and weight o Assess for thrill and audible bruit and signs of inflammation o **Alert of personnel to avoid using the arm with the vascular access site**No blood or BP After Dialysis: o Monitor for orthostatic hypotension, tachycardia and weight loss o Monitor BUN, serum creatinine, serum electrolytes and hematocrit o Report muscle cramping, headache, nausea and vomiting, altered levels of consciousness, seizures or hypotension o If a transfusion was given during dialysis, report symptoms of transfusion reaction, such as chills and fever, dyspnea, chest, back or arm pain; hives or itching

What is the underlying cause for a sickle cell patient to have an exacerbation? What activities would you teach to avoid hypoxia and hypoxemia?

Sickle cell disorders are characterized by the presence of abnormal hemoglobin, hemoglobin S (HbS), in the RBC's. The abnormal hemoglobin causes RBC's to deform and become sickle shaped when blood oxygen levels fall. The deformed RBC's clump together and obstruct blood vessels . Events likely to trigger a sickling event include hypoxia, low temperatures, excessive exercise, anesthesia, dehydration, infections or acidosis. To avoid hypoxia and hypoxemia these patients should avoid excessive or vigorous exercise, avoid swimming (especially in cold temperatures), make sure to drink enough water and have sufficient rest. (Burke Chapter 20 p.480)

More KidneyFailure When to give most scheduled meds on dialysis day (text is not quite accurate)think about the concept:

Since dialysis is the removal of excess fluids and waste, I think it would be best to take non essential medications after dialysis. If a patient needs a medication or it's pertinent to take it on a scheduled time that's before dialysis, a different dosage should be prescribed based on the drug and the effects dialysis would have on the therapeutic effect Be mindful of the types of meds. Such as antihypertensives should be held. Daily meds can be given after dialysis.

Chronic Kidney Disease:

Slow, gradual process of kidney destruction which may go unrecognized: o usually staged: CKD 1-4 o Eventually kidneys are unable to do their job and we get ESRD (End-stage renal disease) *Diabetes is leading cause in all age groups o By the time patient reaches stage 4 of CKD, symptoms of uremia develop o When patient reaches ESRD, they have to get a kidney transplant or dialysis

Urinary Calculi (Urolithiasis): Kidney Stones What is it?

Small, hard, mineral deposits that form inside the kidneys; stones are made of mineral and acid salts o Have many causes and can affect any part of the urinary tract o Stones often form when urine becomes concentrated, allowing minerals to crystalize and stick together -stones form when urine contains crystal forming substances such as; calcium, oxalate and uric acid o Really important to hydrate* Increase fluids

Urinary Incontinence Kegal exercises help this type of incontinence:

Stress urinary incontinence.

BENIGN PROSTATIC HYPERPLASIA Surgery: TURP

Surgery: TURP Obstructed prostate tissue is removed using the wire loop of a resectoscope inserted through the urethra Nursing Care Checklist Before surgery: **Inform patient he will return from surgery with a urinary catheter Continuous bladder irrigation= triple lumen catheter -- 1st Lumen- goes in -- 2nd Lumen- goes to drainage bag -- 3rd Lumen- irrigation bag **Pinkish red drainage is normal, should eventually be clearing After surgery: o Assess and manage pain: may experience abdominal cramping due to gas o Maintain accurate I&O, accounting for amounts of irrigating solution used. Irrigant may be counted as intake, or depending on hospital protocol, the amount of irrigant instilled may be subtracted from urinary output to accurately determine the amount of urine output to accurately determine the amount of urine output o Frequently assess catheter patency; record color and character of urine o For the first 24-48 hours, monitor for hemorrhage (frankly bloody urine, large blood clots, decreased urinary output) VS: decreased BP, increased pulse, decreased H&H, anxiety, restlessness- call HCP o Instruct the patient to avoid straining to void or when having a bowel movement: give stool softener · o Encourage fluids: 3 L of fluids daily o Teach Kegel exercises: start and stop urine stream several times during each voiding to increase muscle strength used to stop urine flow and reduce incontinence o Teach patient to empty leg bag every 3-4 hours o Avoid alcohol, especially at night

A patient with a history of UTIs asks what can be done besides taking antibiotics to help prevent them from occurring. What should the nurse recommend to this patient? Select all that apply.

Take extra vitamin C everyday Limit the intake of alcohol Increase intake of water everyday

How can they be prevented?

Teach measures to prevent UTI to all patients, particularly to young, sexually active women. Encourage a generous fluid intake of 2.0 to 2.5 quarts per day, increasing intake during hot weather or strenuous activity. Discuss the need to void when the urge is felt, emptying the bladder every 3 to 4 hours. Instruct women to cleanse the perineal area from front to back after voiding and defecating. Teach them to void before and after sexual intercourse to flush out bacteria introduced into the urethra and bladder. Advise women to avoid bubble baths, feminine hygiene sprays, and vaginal douches that may dry perineal tissues. Wearing cotton briefs and avoiding underwear made of synthetic materials is also helpful. Postmenopausal women may benefit from hormone replacement therapy or estrogen cream. Unless contraindicated, suggest consuming two or more glasses of low-sugar cranberry juice daily. Preventing Utis Cranberry products are effective in preventing UTI, especially in women with recurrent UTIS. Cranberry interferes with the ability of bacteria to attach to epithelial cells in the urinary tract. Juice appears to be more effective than cranberry capsules, particularly when consumed more than twice a day (Brown, 2012). Blueberries contain a compound that prevents bacteria from attaching to the bladder wall to cause cystitis as well. Bilberry (a relative of the blueberry) is also recommended and is available as an herbal extract in capsules. Saw palmetto is another herbal urinary anti-infective used primarily by men with prostate enlargement. Bromelain (an enzyme derived from pineapple) may increase the effectiveness of antibiotic therapy for treating UTIs. Vitamin C supplements can help treat and prevent UTIS. Dietary changes to prevent UTIS include reduced intake of sugar, alcohol, and fat

A 6 year-old is admitted with sickle cell crisis. The patient has a FACE scale rating of 10 and the following vital signs: HR 115, BP 120/82, RR 18, oxygen saturation 91%, temperature 101.4'F. Select all the appropriate nursing interventions for this patient at this time? A. Administer IV Morphine per MD order B. Administer oxygen per MD order C. Keep NPO D. Apply cold compresses E. Start intravenous fluids per MD order F. Administer iron supplement per MD order G. Keep patient on bed rest H. Remove restrictive clothing or objects from the patient

The answers are A, B, E, G, and H. When a patient is in sickle cell crisis, the abnormal RBCs are sickling and sticking together, which blocks blood flow. To help alleviate the RBCs from clumping together and sickling, oxygen and hydration are priority. This will help dilute the blood (hence decrease the sticking of RBCs) and help supply oxygen to the RBCs (remember abnormal RBCs with hemoglobin S are very sensitive to low oxygen levels and will sickle wh en there is low oxygen). In addition, pain needs to be addressed. Opioid medication is the best on a scheduled basis rather than PRN (as needed). Avoid keeping patient NPO unless needed (remember patient needs hydration). Avoid cold compresses (can lead to more sickling) but instead use warm compresses. The patient will need FOLIC ACID supplements to help with RBC creation rather than iron (iron can actually build up in the body and collect in the organs in patients with sickle cell disease). Patients definitely need to be on bedrest, and restrictive clothing or objects (blood pressure cuff etc.) should be removed to help blood flow.

Why is the BP elevated in glomerulonephritis?

The blood pressure is elevated because when glomerular filtration is interrupted, the glomerular filtration rate falls. This activates the RAA system which leads to salt and water retention, and hypertension.

The elderly are prone to becoming anemic. Why is this? What changes make them at high risk?

The elderly are at risk for decreased red blood cell and hemoglobin production mainly due to chronic disease and nutritional deficits. Inadequate iron intake and impaired iron absorption leads to iron deficiency anemia which is common in older adults. Teaching them to increase foods in meats such as beef, chicken, turkey and tuna and vegetables such as spinach, broccoli, dried beans can increase their daily iron intake. manifestations of iron deficiency are brittle spoon shaped nails, cheilosis-cracks at the corners of the mouth, a smooth sore tongue and pica- a craving to eat unusual non food items such as clay or starch. Another nutritional anemia older adults are at risk for is folic acid anemia. This is commonly due to being malnourished. many older adults have a hard time performing ADL's for themselves as well as knowing the right foods to eat to get their daily intake of necessary nutrients. increasing daily intake of fortified grain products like oatmeal or pasta and eating green vegetables like asparagus, broccoli and spinach aids in supporting their folic acid values. The manifestations of this are diarrhea, cheilosis and glositis along with the symptom's of anemia. Older adults are also at risk for having chronic diseases such as kidney disease and rheumatoid arthritis. They usually go into the hospital with one problem and find out they are anemic while being treated for their main problem. This type of anemia builds gradually, causing fatigue and hypoxia. This makes it difficult for any person to have enough energy to perform ADL'S. Teaching the older adult how to conserve energy, such as getting a shower chair to rest on while taking a shower is a way in keeping them safe. Anemia causes hypoxia which makes blood from tissues go to other parts of the body (organs) to compensate for the lack of rbc and hemoglobin this Increases dizziness,dyspnea, weakness and fatigue.

What is the correlation between someone with chronic kidney disease and having anemia? What labs are you watching for carefully?

The kidneys play a role in the production of red blood cells; erythropoietin is made by the kidneys in response to hypoxia, it stimulates the bone marrow to produce red blood cells. Chronic kidney disease destroys functional kidney tissue and eventually the kidneys can no longer excrete enough erythropoietin to maintain normal RBC levels; this results in anemia. The labs you should watch for carefully are serum creatinine and BUN levels, RBC count, and hemoglobin and hematocrit levels. You would monitor serum creatinine and BUN levels to determine kidney function in a patient with chronic kidney disease. A CBC helps determine the RBC count and hematocrit and hemoglobin levels. RBC count should be 4.6-6.0 million/uL in men and 4.0-5.0 million/uL in women. Normal hemoglobin lab values are 13.5-18 g/dl in men and 12-15 g/dl in women. Normal hematocrit values are 40-54% in men and 36-46% in women. If the patient has anemia, the RBC count, hemoglobin values, and hematocrit values will be decreased. (Burke pg. 467,480, 729, 768, and the Nursing Central App)

What assessment findings would you expect to see in a sickle cell child having an acute exacerbation?

The most common finding of a patient of any age during a sickle cell exacerbation is pain. Children usually exhibit pain in areas of growth such as the long bones. The sickled cells maintain their presence in the marrow of the growing bones during childhood. During adolescence the pain may become more prominent in vertebral areas such as the lumbar region of the back. Other common findings: -Hypoxemia related to respiratory complications. -Body temperature fluctuations. -Anemia. -When sickling occurs inside the chest acute chest syndrome can occur. The cells stick together and block the flow of oxygen. -Jaundice. An acute exacerbation is predominantly marked by an abrupt onset of intense pain, often in the abdomen. However, the pain can be almost anywhere in the body, including the chest, back, or joints. Manifestations of anemia may also be present, such as becoming jaundiced due to bilirubin being released. Other acute events can be sequestration crisis (pooling of blood in the liver or spleen), temporary loss of bone marrow function leading to aplastic anemia, or hemolytic crisis (rapid drop in hemoglobin levels). Reference: Burke, p. 480-81

In a patient with a Hgb of 7 and Hct of 26, what physical assessment findings would you expect to see?

The normal levels for hematocrit are 35-50% and the normal hemoglobin levels are 12-18mg/dl. This patient has significantly low values for both. Low hemoglobin levels indicate anemia. They also mean that the oxygen-carrying capacity of the blood is low, which leads to tissue hypoxia. During a physical exam, a nurse would take note of tachypnea, tachycardia and dyspnea on exertion. Skin, mucus membranes, conjunctiva and nail beds will become pale. Tissue hypoxia also causes angina, fatigue and night cramps. Poor oxygen to the brain causes dim vision, headache and dizziness. Low hematocrit indicates anemia as well, but is also seen in leukemia. The manifestations of leukemia result from anemia and are; pallor, fatigue, tachycardia, malaise, lethargy and dyspnea. The severity of all of these manifestations depends on the severity of the disorder. Burke pg 479, 487

A patient has a suprapubic catheter that is draining cloudy, foul smelling urine. What assessment should the nurse be doing in this situation?

The nurse should assess for a suspected UTI Subjective information from the patient and ask of any symptoms of: o Any history of UTI o Any fever or chills to report o Back or abdomen pain o N/V o Fluid intake - diet and nutrition o Any new meds ( antibiotics, anticholinergics) o Are they pregnant o Sexual relationship / or multiple partners, and any history of sexually transmitted disease Objective information o Vital signs o Temperature o General appearance / mental status o Look at insertion site, any redness or discharge o Palpate abdomen or costovertebral for pain or tenderness o Obtain urine specimen - by clamping the proximal drainage tube and directly withdrawal sample from the port using sterile technique. o Additional labs should be ordered WBC and differential, urinalysis and culture, serum glucose. Kozier & Erbs ( Fundamentals of Nursing), page 1202 Burke ( Medical -Surgical Nursing Care), pages 744-746

If a female patient came to the clinic complaining of extreme fatigue to the point where she was unable to carry out her ADLs, what would be your nursing assessment? What significant characteristics would you be looking for?

The nurse will assess a health history of fatigue, chest pains, obvious bleeding (excessive menses, rectal, or vomiting) genetic, previous problems, and current pains. Dietary history is important. A physical assessment including orthostatic bps, nail beds for perfusion capillary refill and spooning of the nail bed, conjuctive for color and presence of petechiae or ulcers, mouth and lips for glossitis (inflammation of tongue) cheilosis ( tounge cracks at edges) observe for dyspnea and SOB with activity, if bleeding suspected check stool and vomit appearance. Also checking the palms for pallor. Lab results for RBC, hemoglobin, and hematocrit. You can follow a nada for Activity Intolerance.Since fatigue is such a common and vague term, as part of your history taking, ask the patient how their functioning compares to 6months to 1 year ago.

What is the nurse's role when someones is experiencing an adverse reactions to a blood transfusion? What are some common adverse reactions? What would the nurse do first?

The nurses role when someone is experiencing a adverse reaction is to 1. stop it immediately and notify whoever is in charge 2. start normal saline slowly 3. take vital signs and assess for signs of a further reaction 4. continue to monitor and treat the reaction according to the physicians orders Some common adverse reactions to a blood transfusion can be: -feeling warm -chills -itching -weakness -fainting -difficulty breathing -fever In the event of a reaction to a blood transfusion the first thing that the nurse should do is STOP the transfusion. (Burke pg 323)

The basic pathophysiology of anemia involves a reduction in:

The total number of circulating erythrocytes

Working with this physician, you see many patient with anemia. The basic pathophysiology of anemia involves a reduction in:

The total number of circulating erythrocytes

What type(s) and route of mediation is given to a patient in sickle cell crisis who is admitted to the hospital?

The type of medication give to a patient in sickle cell crisis who is admitted to the hospital are Pain medication can be orally or parenterally Supplemental iron can be given orally or parenterally Vitamin B12 can be give orally to patients with nutritional vitamin B12 deficiency or parenterally to patients with pernicious anemia Folic Acid can be given orally or parenterally to patients Hydroxyurea is prescribed for patients with severe sickle cell disease, in which promotes fetal hemoglobin production. Aplastic anemia is treated with immnosuppressive therapy or androgens to stimulate blood cell production. Blood transfusion is another treatment for sickle cell disease IV Hydration (Burke pg 482) Hydroxyurea treats sickle cell by helping to prevent formation of the sickle cell red blood cell.

Folic acid, vitamin B12 and thiamine are often given to individuals with alcohol dependence, why is this and what type of anemia are they at risk for?

They are chronically malnourished (inadequate dietary intake) they are at risk for Folic Acid deficiency anemia. This malnourished state causes them to be deficient in folic acid, thiamine, and b12. Like vitamin b12, folic acid is required for normal production and maturation of RBCs. Burke p.479-480

What would a patient with thrombocytopemia exhibit? Include labs, physical assessment findings and potential complications.

Thrombocytopenia is a condition where you have a platelet count of less than 100,000 platelets/mL of blood. It is the most common cause of abnormal bleeding most commonly from decreased platelet production, increased destruction of platelets, or accumulation of platelets in the spleen in children and adults (20-40 years old more so women than men), inherited autoimmune disease/disorder (Lupus/infections/HIV/hepatitis), and side effect of medication (heparin). A CBC with platelet count lab would be ordered; normal range is 100,000 or higher. Anything < 100,000 platelets/mL would require in-depth evaluation with a bone marrow examination or a serologic testing when heparin-induced thrombocytopenia (HIT) is suspected. Physical assessment findings: Purpura (easy bruising) Ecchymoses Petechiae (small, flat, purple, or red spots on skin) Prolonged bleeding from minor cuts, gums, or nose Other bleeding of heavy menstrual cycle, hematuria, or GI bleeding (rectal/tarry stools) Headache, weight loss, fever, and fatigue Monitor LOC and vital changes (look for hypotension and tachycardia) Potential Complications: <20,000 platelets/mL can indicate high alert internal or external bleeding. Bleeding in the brain (rare) can be fatal.

Peritoneal Dialysis:

Tube inserted through abdomen: Patient can complete at home o Make sure to monitor for Peritonitis o patients at risk for developing glucose issues and weight gain **If you see poor dialysate drainage: lower the bag; reposition the patient and check tubing for kinks, remember to take VS

Testing:

UA **STRAIN ALL URINE- If kidney stones expected KUB to identify calculi- x-ray of kidney/ureter/bladder IVP, Renal US, CT scan or MRI to locate calculi Cystoscopy to visualize and possibly remove stones

More Kidney Failure What is uremia? How does it affect the brain? What can you expect to see after dialysis?

Uremia is the accumulation of toxic waste products in the body. It has an effect on mental status including: difficulty thinking, concentrating, fatigue and insomnia. I would expect to see a decrease of toxic waste in the body since dialysis will remove those excess products.

During shift report, the nurse learns that an older female client is unable to maintain continence after she senses the urge to void and becomes incontinent on the way to the bathroom. Which nursing diagnosis is most appropriate?

Urge Urinary Incontinence Rationale: The key phrase is "the urge to void." Option 1 occurs when the client coughs, sneezes, or jars the body, resulting in accidental loss of urine. Option 2 occurs with involuntary loss of urine at somewhat predictable intervals when a specific bladder volume is reached. Option 3 is involuntary loss of urine related to impaired function.

Urinary Calculi When stones are too big, this procedure is performed:

Urinary Calculi lithotripsy

A 6 hour post surgical patient is not voiding. What are some causes for the urine retention? How would the nurse assess for bladder retention? And what are nursing interventions that could be done to address this problem?

Urinary retention is when the normal bladder emptying function is disrupted, when the bladder can't be emptied it becomes overstretched. This effects detrusor muscle contractions and further impairs urination. Surgery itself (especially abdominal or pelvic surgery) can affect detrusor muscle function which leads to urine retention. Other causes of post surgical urine retention include the effects of anesthesia, use of narcotics, pain, inactivity, altered fluid balance, and anxiety. Acute urine retention can lead to acute kidney injury and urinary tract infection so it's very important to carry out nursing interventions to resolve the retention and prevent these complications from occurring. Nursing interventions to improve urine retention include assessing for bladder distention and obtaining a bladder scan to verify that there is retained urine, increasing oral fluid intake to 2500-3000 ml to facilitate renal function, running water in the sink or shower while the patient tries to void, placing the patient's hands in warm water while they are trying to void, pouring water over the perineum while sitting on the beside commode or toilet, and assisting the patient with using a bedpan, bedside commode, or walking to the bathroom (if able) to promote normal urinary elimination. If the previous interventions fail to treat the urine retention, and after obtaining an order, the bladder may be emptied through straight catheterization.

The patient admitted to the intensive care unit after a motor vehicle accident has been diagnosed with AKI. Which finding indicates the onset of oliguria resulting from AKI?

Urine output less than 400 mL for the past 24 hours The most common initial manifestation of AKI is oliguria, a reduction to urine output to less than 400 mL/day.

A nurse is assessing a patient who is at risk for developing acute kidney injury. The nurse would become most concerned if which of the following assessments was made?

Urine output of 20 mL/hr for the last 3 hours, serum BUN elevated and serum creatinine elevated

Urinary Calculi (Urolithiasis): Kidney Stones Calcium Stones: most common:

Usually in the form of calcium oxalate. Oxalate normal substance found in foods, peanuts, chocolates

What types of foods would you recommend to an individual with iron deficiency anemia and is vegetarian?

Vegetarians being people who restrict their diets to vegetables are not getting enough iron due to the fact that meats, poultry and fish provide readily absorbed iron to the body. Iron contained in vegetables and grains are less readily absorbed. I would encourage them to eat foods high in Vitamin C in their diets to increase the absorption of iron from grains, legumes and other sources. In addition to their vegetarian diets, foods rich in vitamin C are as follows; citrus family (orange juice or orange, lemon, tangerine etc), tomatoes, strawberries, papaya, kiwi, kale, broccoli, sweet potatoes, red, green and yellow pepper, cantaloupe, cauliflower etc. Burke 482, 484.

An intravenous pyelogram (IVP) is scheduled. You determine that the patient is knowledgeable about this procedure when he states it is being done to: renal colic and nephrolithiasis who has lithotripsy.

Visualize and locate any stones in the urinary tract.

A patient is just started on long term warfarin (coumadin) therapy. Many patients get vit K and potassium mixed up. What would the nurse include in the teaching?

Vit K is used as an antidote for warfarin (coumadin) . When taking warfarin (coumadin) limit your intake of foods rich in vitamin K such as asparagus, beans, broccoli, brussels sprouts, cabbage, cheeses, fish ,greens, milk, pork, rice spinach, turnips, and yogurt. Use a soft toothbrush and electric razor. Potassium is used to prevent and treat hypokalemia. Take potossium supplements with meals to reduce gastric irritation. Take as prescribed, do not use salt subsititues when taking potassium. Burke pg 125, 454

CBC values

WBC: 4.2 - 11.0 K/mcL RBC: 4.00 - 5.20 mil/mcL HGB: 12.0 - 15.5 g/dL HCT: 36.0 - 46.5 % MCV: 78.0 - 100.0 fl MCH: 26.0 - 34.0 pg MCHC: 32.0 - 36.5 g/dL RDW-CV: 11.0 - 15.0 % PLT 140 - 450 K/mcL NRBC <=0 /100 WBC Absolute Neutrophils 1.8 - 7.7 K/mcL Absolute Lymphocytes 1.0 - 4.0 K/mcL Absolute Monocytes 0.3 - 0.9 K/mcL Absolute Eosinophils 0.1 - 0.5 K/mcL Absolute Basophils 0.0 - 0.3 K/mcL Absolute Immmature Granulocytes 0.0 - 0.2 K/mcL RDW-SD 39.0 - 50.0 fL

Anemia of Chronic Disease

What is it? Patients of chronic diseases such as HIV infection, rheumatoid arthritis, inflammatory bowel disease, chronic hepatitis, and chronic kidney disease often have mild to moderate anemia; depends on the severity of the underlying disease. -Treat underlying disease, get it under control and that will help the anemia that's gone along with it. -If symptoms become severe a blood transfusion or injection of synthetic erythropoietin (a hormone that's normally produced by the kidney) helps stimulate RBC production and ease fatigue* Anemia of Chronic Disease Hemolytic Anemias: Sickle cell Disorders Aplastic Anemia

Iron Deficiency Anemia

What is it? Caused by an inadequate supply of iron for RBC formation; most common type of anemia - o Body fat make hemoglobin without iron. - o Iron deficiency leads to fewer RBCS, microcytic (small) RBCS and hypochromic (pale) RBCS Risk Factors: o Older adults o Pregnant women o Excess blood loss from adults, can also lead to iron deficiency anemia o Menstrual blood loss in adult females o Chronic, occult (hidden) blood loss: examples: bleeding ulcers or hemorrhoids Chronic Iron Deficiency Anemia Manifestations: o Brittle, spooned-shaped nails o *Cheilosis (cracks at the corners of the mouth) o Smooth, sore tongue o *Pica (a craving to eat unusual substances, such as clay or starch*) Causes: o Poor diet o Impaired absorption o Increased need for the nutrient Diagnostic Tests: o CBC o Iron Levels: **Oral contraceptives can affect results** o TIBC (total iron-binding capacity tests): -o Measures the maximum amount of iron that can bind to transferrin, the protein that transports it o Serum Ferritin: Measures the amount of iron stored in body tissues Giving Medications Safely: Iron: (Ferrous sulfate and Ferrous gluconate) o Side of effect of Feosol (ferrous sulfate) is nausea o Make sure to take Feosol (ferrous sulfate between meals) o Give with orange juice to improve absorption o Administer elixirs through a straw to prevent staining of teeth o Use Z-track technique because its staining to tissue - o Administer as Deep IM -o Change needle before injection -o 90 degree angle: 1.5 inch needle used -o Displace skin on ventrogluteal -o Hold for 10 seconds, then release o Monitor hemoglobin and RBC counts Patient Teaching: o Take with food (not milk) to reduce gastric distress o Stools may be dark green or black; this is harmless* o Increase fluid and fiber intake to decrease constipation o Eat spinach, *broccoli*, red meats, beef liver, chicken, turkey, dark chocolate Interdisciplinary Care: Medications Dietary modifications Blood transfusions

Sickle Cell Anemia

What is it? Sickle cell disorders are characterized by the presence of abnormal hemoglobin, hemoglobin S (HbS), in the RBCS. Life long disease and varies person to person Genetically transmitted/ Inherited: Abnormal hemoglobin S (HbS) Sickle cell anemia or sickle cell trait Hypoxia→ sickle shaped cells Repeated episodes → damage RBC membranes s/s Sickling→ intense pain Oxygen supply affected→ sickled RBC's→ obstruct blood flow and cause cellular hypoxia→ destroy organs and tissues Anemia and jaundice Genetic counseling Diagnostic Tests: Sickle cell screening test (identify Hgb S): Hemoglobin electrophoresis Treatment: Meds: Folic acid supplements & Hydroxyurea Blood transfusions Genetic counseling Pain medication IV hydration Elevate HOB Keep room warm Assess circulation & for s/s infection Normal red blood cells are round. In sickle cell anemia, some red blood cells become deformed, so they look like a sickle used to cut wheat. These unusually shaped cells give the disease its name. With each pregnancy, two people with sickle cell traits have: o A 25 percent chance of having an unaffected child with normal hemoglobin o A 50 percent chance of having a child who also is a carrier o A 25 percent chance of having a child with sickle cell anemia o People who have sickle cell disease inherit two abnormal hemoglobin genes, one from each parent. - o In all forms of sickle cells disease at least one of the two abnormal genes causes a person's body to make hemoglobin S. - o Sickle Cell Anemia is when a person has two hemoglobin S genes, hemoglobin SS (most common and most severe kind of sickle cell disease) o (Know what you would say if you had a question with a parent wondering about it, whether their children will have it) o Sickle Cell hemoglobin is not normal hemoglobin; it forms stiff rods within the red cell changing it into a crescent or sickle shape. Sickle shape cells are not flexible and stick to vessel walls causing a blockage that slows or stops the flow of blood. When this happens oxygen can't reach nearby tissues. o The lack of oxygen can cause attacks of sudden severe pain called Pain Crisis: These pain attacks occur without warning and a person often needs to go to the hospital for effective treatment. o The spleen and kidneys are especially prone to sickling damage, along with other organs and joints *Spleen is especially prone to serious bacterial infections that can be life-threatening o Sickle cells last only 10-20 days: body may have trouble keeping up with how fast the cells are being destroyed and because of this; the number of blood cells is usually lower than normal: this condition is called anemia and can make a person have less energy Manifestations: o Painful swelling of the hands and feet o Fatigue or fussiness because their anemic o Yellowish color of skin or eyes (Jaundice): Occurs when large number of RBCS hemolysis* Pain Crisis: People describe as o Sharp, intense, throbbing or throbbing (can be worse than post-surgical or birth pain) o Strikes anywhere in the body in more than one spot at a time o Normally occurs in lower back, legs, arms, chest and abdomen Diagnostic Tests: *Hemoglobin Electrophoresis: used to identify abnormal forms of hemoglobin when a generic anemia such as sickle cell is suspected Sickle cell screening test Risk Factors: o Most common in African Americans o Babies can be a carrier and not have disease o Always present at birth o Illness can cause pain crisis to occur, extreme temperature changes, stress, dehydration, high altitudes o *Important to advice patient to drink lots of fluids o *People with sickle cell can have severe anemia and at times can be so severe its life threatening Nursing Care for Patients: Prevent the crisis, try to be stress life, try to prevent infections, drink lots of fluid, watch temperature changes, eat a well-balanced diet, exercise, avoid smoking and alcohol Medications: Hydroxyurea o -Reduces and prevents several sickle cell complications: Known to increase the amount of hemoglobin F. Increased hemoglobin F provides protection against the effects of hemoglobin S. o Should be taken regularly o Careful monitoring for blood testing dose adjustments o *Children should take penicillin prophylactically twice a day to reduce infections like pneumonia and *avoid pain crisis o Should receive vaccines to prevent any further infections

Pernicious Anemia (Vitamin B12)

What is it? Vitamin B12 is a nutrient found in some foods. The body needs this nutrient to make healthy RBCS and to keep the nervous system working properly o Vitamin B12 deficiency interferes with the maturation of RBCS. Great numbers of large, immature RBCS move into circulation, become fragile and incapable of carrying oxygen in adequate amounts Causes: • o Caused by impaired absorption of vitamin B12 in the GI tract, intrinsic factor secreted by the GI tract binds with vitamin B12 so it can be absorbed by the body. -Intrinsic factor is a protein made in the stomach, a lack of this protein leads to pernicious anemia -Without intrinsic factor, Vitamin B12 can't be absorbed and leaves the body as a waste product -Lack of intrinsic factor may be due to an autoimmune reaction in which the immune system is mistakenly attacks stomach cells that produce it o Vitamin Bl12 deficiency can also occur if the small intestine can't absorb Vitamin B12; - may be due to surgery to the stomach or small intestine (ex. gastric bypass surgery) • o Abnormal bacterial growth in the small intestine • o Crohn's disease or Celiac's Disease- interferes with absorption of vitamin Manifestations: o *Smooth, sore, beefy red tongue • o *Diarrhea Essential for neurological function; may experience • o Paresthesias (altered sensations, such as numbness or tingling) in the extremities o Problems with Proprioception (the sense of one's position in space) Can't prevent pernicious anemia caused by lack of intrinsic factor, although uncommon, some people develop pernicious anemia because they are not getting enough Vitamin B12 in their diet Steps to prevent caused by dietary factors: Vitamin B 12 required for RBC formation & maturation Pernicious Anemia Intrinsic factor Signs and symptoms: Sore, beefy red tongue (glossitis) Diarrhea Paresthesias Problems with proprioception Diagnostic Tests: Schilling's Test, Serum Vitamin B12 and Folic Acid (Folate) levels Treatment: Vitamin B12 injections, Vitamin B 12 po

Aplastic Anemia

What is it? When the bone marrow fails to produce RBCS. o Normal for blood cells to die* o RBCS- live about 120 days o WBCS- live less than a day o Platelets- live about 6 days -If bone marrow can't make enough new blood cells many health problems can occur; irregular heartbeat, enlarged heart, infections and bleeding Severe aplastic anemia can even cause death -Is a rare, but serious disorder and can develop suddenly or slowly. Disorder tends to get worse over time unless its cause is found in treatment. -Aplastic anemia may follow injury to stem cells in bone marrow caused by radiation, certain chemicals (pesticides, arsnic), and an immune response. When damaged they can't grow into healthy cells. Causes: o -Infectious diseases o -Mono o -HIV Treatments: • o Blood transfusion o Blood and marrow stem cell transplant o Medicines Diagnostic Tests: o Hematocrit and Hemoglobin- Low o RBC- Low (also known as pancytopenia) o Platelet-Low- Increases risk for bleeding o WBC- Low (also known as pancytopenia) Bone Marrow Aspiration/ Biopsy Nursing Care: o Need informed consent- know risks and benefits o Need VS o Place in supine or prone position o **Tell patient to remain still o Takes 20 minutes start-finish o **Monitor for bleeding/infection/pain (apply pressure 5-10 minutes) o Takes 20 minutes from start-finish o Site may ache for several days o Allow for periods of rest when client is fatigued- Due to activity intolerance o Monitor vital signs / oxygenation status / labs o Refer to dietitian if is diet is deficient o Teach important points about meds o Monitor risk for falls- complaints of dizziness Bone marrow fails to produce RBC's Injury to stem cells caused by radiation, chemicals, chemotherapy, certain antibiotics Pancytopenia: decrease in RBC's, WBC's and platelets Signs/ symptoms: RBC deficiency→ Platelet deficiency→ WBC deficiency→

Dialysis

a procedure to remove waste products from the blood of patients whose kidneys no longer function many meds are given after dialysis so that they do not get filtered out. This is true for many HTN and cardiac meds.

Percutaneous Lithotripsy-

another surgery available

The nurse caring for a patient with a diagnosis of thrombochtopenia knows that this disorder will affect the patient's: a. ability to form blood clots. b. ability to fight infection. c. energy level. d. nutritional status.

answer: a

Urinary Calculi (Urolithiasis): Kidney Stones Manifestations

are due to obstructed urine flow: Symptoms can change as kidney stones are moving

Important nursing interventions for the patient with AKI are (select all that apply)

careful monitoring of intake and output. daily patient weights. meticulous aseptic technique. frequent mouth care. You have an important role in managing fluid and electrolyte balance during the oliguric and diuretic phases of AKI. Observing and recording accurate intake and output are essential. Measure daily weights with the same scale at the same time each day to assess excessive gains or losses of body fluids. Mouth care is important to prevent stomatitis, which develops when ammonia (produced by bacterial breakdown of urea) in saliva irritates the mucous membrane.

Measures indicated in the conservative therapy of CKD include

decreased fluid intake and protein intake; increased carbohydrate intake. Water and any other fluids are not routinely restricted in the pre-end-stage renal disease (ESRD) stages. Patients on hemodialysis have a more restricted diet than patients receiving peritoneal dialysis. For those receiving hemodialysis, as their urinary output diminishes, fluid restrictions are enhanced. Intake depends on the daily urine output. Generally, 600 mL (from insensible loss) plus an amount equal to the previous day's urine output is allowed for a patient receiving hemodialysis. Patients are advised to limit fluid intake so that weight gains are no more than 1 to 3 kg between dialyses (interdialytic weight gain). For the patient who is undergoing dialysis, protein is not routinely restricted. The beneficial role of protein restriction in CKD stages 1 through 4 as a means to reduce the decline in kidney function is being studied. Historically, dietary counseling often encouraged restriction of protein for CKD patients. Although there is some evidence that protein restriction has benefits, many patients find these diets difficult to adhere to. For CKD stages 1 through 4, many clinicians encourage a diet with normal protein intake. However, you should teach patients to avoid high-protein diets and supplements because they may overstress the diseased kidneys.

The nurse is giving discharge instructions to a patient treated for renal calculi. The nurse explains that the stones can be prevented in many cases by:

drinking 8 to 10 glasses of water daily.

In a severely anemic patient, the nurse would expect to find:

dyspnea and tachycardia

Urinary Calculi (Urolithiasis): Kidney Stones Struvite Stones:

form in response to infection such as an UTI. These stones grow quickly and become quite large

During the oliguric phase of AKI, you monitor the patient for (select all that apply)

hypertension. electrocardiographic (ECG) changes. pulmonary edema. You monitor the patient in the oliguric phase of AKI for hypertension and pulmonary edema. When urinary output decreases, fluid retention occurs. The severity of the symptoms depends on the extent of the fluid overload. In the case of reduced urine output (anuria and oliguria), the neck veins may become distended and have a bounding pulse. Edema and hypertension may develop. Fluid overload can eventually lead to heart failure, pulmonary edema, and pericardial and pleural effusions. The patient is monitored for hyponatremia. Damaged tubules cannot conserve sodium, and the urinary excretion of sodium may increase, resulting in normal or below-normal levels of serum sodium. Monitoring may reveal ECG changes and hyperkalemia. Initially, clinical signs of hyperkalemia are apparent on electrocardiogram, which demonstrate peaked T waves, widening of the QRS complex, and ST-segment depression. Urinary specific gravity is fixed at about 1.010.

The nurse is preparing a patient recovering from sickle cell crisis for discharge. To prevent future crises from occurring, the nurse should instruct the patient to:

keep well hydrated.

Anemia 3 main causes of Anemia:

o 1. Blood loss o 2. Lack of RBC production o 3. High rate of RBC destruction o May be the result of diseases, conditions or other factors o **No matter the causes, the end result is the same and that's that the blood has a decreased amount of oxygen carrying capacity and it results in tissue hypoxia. o **Tissue Hypoxia may also cause angina or dyspnea on exertion o **Kidneys release increased amounts of erythropoietin, which stimulates bone marrow, causing bone pain o **Blood is redistributed to vital organs causing pallor of skin, mucous membranes, nail beds, and conjunctiva ----Body knows enough to take away from the periphery and get to the main core to keep body going o *These signs and symptoms occur because the heart has to work harder to pump oxygen riched blood through the body o -Some can be treated with diet or dietary supplements o -Some can be life threatening if not diagnosed and treated o Severity of the manifestations of anemia depend on the cause and the severity of the disorder. For example: rapid blood loss causes immediate symptoms, whereas the person with slowly developing anemia may have no symptoms until the condition is advanced or if the oxygen needs of the body increase. o Vitamin Deficiencies usually develop slowly over several months to years. Disorders of the RBC's affect oxygen to tissues Anemia: too few RBC's Causes of Anemia: Impaired RBC formation Excessive loss of RBC's Destruction

Anemia Blood is made up of many parts including;

o 1. RBCS- carry oxygen and remove carbon dioxide away from body, made in bone marrow o 2. WBCS- Fight off infection- made in bone marrow o 3. Platelets-Stops bleeding and seals cuts- made in bone marrow * Patients with anemia will have low levels of all three blood cells* Red blood Platelets White blood cells cell -If a patient has anemia they do not have enough oxygen riched blood and as a result patient will feel weak and/or tired- FATIGUED - Don't have energy to do normal activities

Glomerular Disease:

o A capillary in a disease glomerulus lets protein into the urine (proteinuria) and red blood cells into the urine (hematuria). -can be acute or chronic **Acute occurs following a strep infection (strep throat or strep infection on skin) -Occurs 10-14 days after the initial strep infection **Chronic is usually the result of kidney damage by diabetes; symptoms develop slowly and is often unrecognized until the patient is exhibiting signs of renal failure o Laboratory analysis of the urine often show protein and blood in the urine o When glomerular filtration is disrupted the GFR falls and azotemia (increased blood levels of nitrogenous wastes, including urea and creatinine) occurs. o When GFR falls, is activates the renin-angiotensin-aldosterone, leading to increased salt and water retention, increasing our blood pressure (hypertension).

MANIFESTATIONS OF END-STAGE RENAL DISEASE AND UREMIA

o Anorexia, nausea, vomiting o Difficulty thinking, concentrating, fatigue, insomnia o Edema, high blood pressure o Restless leg syndrome, numbness, tingling of hands and feet o Dry, itchy skin; pallor and yellowish skin color o Irregular menses, impotence o Anemia with fatigue, weakness, depression o Bone tenderness, pain, muscle weakness o Electrolyte imbalances (high potassium, phosphate, and magnesium levels; low calcium levels) o Metabolic acidosis leading to increased respiratory rate and depth, malaise, weakness, headache

Urinary Retention Nursing care

o Bladder scan and check urine residual o Cholinergic Medications: promote bladder emptying o Crede method (applying pressure over the symphysis pubis with the fingers of one or both hands o Valsalva Maneuver: also promotes bladder emptying o Use trigger points: Used for neurogenic bladder stroking or pinching the abdomen, inner thigh, or glans penis o Intermittent straight cath: May be done at home ---Less change of infection than in indwelling catheter.

Nursing Care Checklist:

o Bowels need to be cleaned out o Asses urine, expect hematuria for 12-72 hours after surgery o Instruct patient to report drainage of urine from incision for more than 4 days

BENIGN PROSTATIC HYPERPLASIA Characteristics of Normal Urine on Urinalysis

o Color--> Light straw to amber yellow; clear o Odor--> Aromatic (Odor should not be pungent: Aromatic) o Specific gravity 1.005-1.030 o pH 4.5-8.0 o Protein -->Negative to trace (0-5 mg/dL) o Glucose -->Negative o Ketones -->Negative o RBC--> 1-2/low-power field (LPF) o WBC--> 3-4/LPF o Casts --> Negative to occasional o Bacteria --> Negative

Glomerulonephritis What do kidneys do for us?

o Control the balance of water in the body o Tell the body when to start and stop making oxygen-carrying red blood cells o Control how acidic the blood is o Filter the blood and pass the waste products on to the bladder as urine o Control blood pressure

Manifestations Glomerulonephritis

o EDEMA o Albuminuria (Proteinuria)- frothy urine o Hypoalbuminemia- Loss of protein in blood because coming out of urine o Hyperlipidemia- High lipids in blood due to loss of albumin in kidneys o Thromboemboli- form blood clots easily o Children more likely to recover completely; adults may have persistent problems leading to kidney failure

Glomerulonephritis When having kidney problems what do we see?

o Edema o Decreased red blood cells, which causes anemia o Acidosis in the blood o Accumulation of waste products in the blood called uremia o Hypertension Nephropathy: Any disease of the kidney Nephrosis: Any degenerative kidney disease without inflammation Nephritis: Inflammation of the kidneys Nephron: Working structure of the kidney

Chronic Kidney Disease: Food and Fluid Management:

o Fluid and Sodium Restriction o Accurate I &O o Avoid salt substitutes because of increased levels of potassium o Protein intake may be limited o Carbs are increased for calories- High Carb o TPN if patient is unable to eat

Manifestations of Acute Glomerulonephritis

o Hematuria; cola- or coffee-colored urine o Proteinuria Frothy urine (foamy) o Edema: periorbital and facial; dependent (upper extremities in particular) o Hypertension o Fatigue Anemia o Anorexia, nausea, vomiting o Headache o Elevated BUN and serum creatinine Know

Urinary Retention Complication

o Hydronephrosis: Urine backs up into the kidneys leads to kidney damage or failure o Acute kidney injury o UTI

RISK FACTORS FOR UTI

o Instrumentation of the urinary tract (e.g., catheterization, cystoscopy) o Structural abnormalities, obstructions, or strictures o Incomplete bladder emptying o Chronic diseases such as diabetes (hyperglycemia can also lead to glucose in the urine and bacteria like it nice and sweet.) Females o Short, straight urethra o Proximity of urinary meatus to the vagina and anus o Tissue trauma and possible contamination during sexual intercourse o Use se of a diaphragm for birth control o Personal hygiene practices Males o An enlarged prostate gland Older Adults o More alkaline (less acid) urine o Higher incidence of diabetes and glucosuria (glucose in the urine promotes bacterial growth) o Incomplete bladder emptying, urinary retention o Changes in vaginal pH in women; decreased prostatic secretions in men The bladder mucosa becomes inflamed and congested with blood pus forms & bleeding may occur

Other types of Anemia known as: Anemias

o Iron poor blood o Tired blood o Low blood RBC destruction-sickle cell, enzyme deficiency Secondary: Bleeding, leukemia, cancer or chronic kidney disease Erythroblastosis Fetalis: destruction by antibodies decrease B12: Pernicious Anemia Lack of intrinsic factor Hypochromic: iron or vitamin deficiency Aplastic: malfunctioning bone marrow

Urinary Tract Infections PREVENTION!

o Monitor urinary output o Provide easy access for toileting o Encourage fluid intake, unless contraindicated, avoid caffeine o Teach patient to be compliant with medications o Empty bladder every 2-4 hours while awake o Remove catheter as soon as possible! o Hygiene- wiping front to back o Voiding before and after intercourse o Providing good catheter care- sterile technique when inserting o Increase fluids- cranberry juice o Eat yogurt o Seek health early o Cotton underwear better for breathing o When bathing no bubble baths, showers are better, watch soaps o Make sure to eat, rest and decrease Increase intake of water everyday Take extra vitamin C everyday Limit the intake of alcohol

Cystoscopy: Patient Teaching

o Needs informed consent o Procedure takes about 30-45minutes. Local or general anesthesia used o May feel pressure or an urge to urinate as the scope is inserted o Should not attempt to stand up without assistance (dizziness or faint) o Burning on urination for a day or two after the procedure is considered normal o Contact doctor immediately if you have bright red bleeding, low urine output, abdominal or flank pain, chills or fever o Warm sitz bath and medications help relieve discomfort o **Increase fluid intake to decrease pain, ease voiding, and reduce the risk of infections o Laxatives may be ordered to prevent constipation and straining after the procedure

MANIFESTATIONS OF BENIGN PROSTATIC HYPERPLASIA

o Nocturia (early symptom)---- 2 or more times at night o Diminished force of urinary stream----trickle o Hesitancy in starting voiding o Dribbling after voiding o Incomplete bladder emptying o Frequency, urgency o Urge incontinence o Dysuria, hematuria

Urinary Retention MANIFESTATIONS

o Overflow voiding or incontinence may occur (constant dribbling) o Assessment reveals a firm, distended bladder o Ask patient to void first, supine position with knees bent slightly, palpate in between the umbilicus and pelvic area.

Kidney Failure Potential Complications:

o Respiratory: Fluid buildup in lungs: causing shortness of breath, chest pain, muscle weakness, elevated potassium levels in the blood are particularly dangerous. o End-Stage Renal Failure o Death

Anemia Patient may also have other symptoms such as:

o SOB o Dizziness- Due to poor oxygen delivery to brain o Headaches- Due to poor oxygen delivery to brain o Constipation- decreased bowel sounds o Muscle weakness o Tachycardia Severe long lasting anemia can damage the heart, brain and other organs in the body and severe anemia can result in death

Glomerulonephritis Medications and Nursing Care: Relieving Symptoms:

o Strep infection: give penicillin ABT o High blood pressure: give anti-hypertension - o Inflammation: give glucocorticoids *Avoid nephrotoxic drugs (some antibiotics, dye, and NSAIDS, aspirin (blood thinners) o Plasmapheresis o Sodium restriction o Dietary Proteins *Increase protein intake if you have proteinuria *Decrease protein intake if you have azotemia o May order a 24-hour urine o At start time, have patient empty bladder completely and discard urine o Save all urine for next 24 hours in the container, placing container on ice or refrigerating as indicated. If any urine is missed, the collection must be restarted o At the end of collection time, have patient empty the bladder completely and save this urine in the container. o Looking for proteins in urine o UA o Throat/ skin cultures for Strep o BUN & Creatinine o EGFR and creatinine clearance o Lytes o Renal US / kidney scan / biopsy

Chronic Kidney Disease: Collaborative Care:

o Tests - blood and urine o Kidney Biopsy

Nursing Care Checklist: Urinary Catheterization

o Use intermittent catheters than indwelling to reduce the risk of infection o Use an appropriate size catheter, not to small because it will leak, not to big because it will stretch out those muscles. o Provide perineal care before cath, use soap and water, rinsing carefully o Do not drain more than 750-1,000 mL from the bladder at one time o Assess urine, color, clarity and amount o Monitor I&O o Make sure bag is below the bladder

Oral iron supplements are prescribed for a 6-year-old child with iron deficiency anemia. To promote optimum absorption, the nurse instructs the mother to administer the iron with:

orange juice.

A patient is admitted with a diagnosis of pernicious anemia. Priority nursing care would include: 1. preventing infection 2. increasing iron intake 3. providing rest periods 4. increasing fluid intake

patient with anemia require frequent rest periods until correction of process can be accomplished.

A patient with extreme fatigue is being interviewed by the nurse. The patient reports a sore tongue and frequent bouts of diarrhea and propriocetion. The nurse should suspect that the patient is experiencing:

pernicious anemia.

The nurse is caring for a patient with thrombocytopenia. The nurse expects that the patient might be treated with:

platelet transfusion.

An patient is admitted with acute glomerulonephritis with symptoms of fatigue, anorexia, and blood pressure of 140/94. The nurse identifies the priority goal of treatment is to:

reduce the hypertension.

patient will be watching the calcium intake in his diet. patient should limit intake of which of the following foods? renal colic and nephrolithiasis who has lithotripsy.

sardines and yogurt

the nurse observes a mother giving an oral iron supplement to her 6-yr old child with iron deficiency anemia. which action by the mother indicates the need for further teaching?

the mother administered the iron with milk. Rational : Milk may affect absorption of the iron. vitamin C increases the absorption of iron by the body. the mother should be instructed to administer the medication with citrust fruit or juice that is high in vitamin C. water will not assis in absoption. (Saunders NCLEX-RN 6th edition)

A patient is admitted to the hospital experiencing respiratory distress. After assessment, the nurse concludes that the patient is in sickle cell crisis. The nurse plans care for the relief of pain because:

the sickled cells accumulate in tissue.= The sickled cells accumulate in the blood vessels causing hypoxia.

More KidneyFailure How does potassium play a role in patients with CKD?

there is excess potassium because electrolytes cannot be excreted, hyperkalemia. hyperkalemia causes muscle weakness, nausea and diarrhea, and dysrhythmias.

BENIGN PROSTATIC HYPERPLASIA Prostate-specific antigen level:

usually higher than normal in BPH- a lot higher in prostate cancer o Schedule test before or at least two weeks after any manipulation of prostate glands; can increase levels

The nurse is caring for a patient who is being treated for anemia and is taking Imferon (Iron) IM. The nurse should plan to administer this medication:

via the z-track method

Urinary Incontinence Due to my enlarged prostate, I have this type of incontinence:

voiding difficulties: urinary distention with bladder over distention and frequent loss of small amounts of urine 25-50ml

Types of Urinary Incontinence Functional

ΤΥΡΕ Functional DESCRIPTION Inability to get to toilet facilities to urinate .Leakage of urine caused by factors other than the lower urinary tract; unable to get to toilet to urinate CAUSE: Self-care deficit Physical disability or impaired mobility Neurologic disorders (AD, CVA) Diuretic therapy or sedation Lack of facilities, privacy or caregiver assistance SYMPTOMS: No urinary symptoms present, only incontinence INTERVENTIONS PATHOPHYSIOLOGY Self-care deficit interferes with ability to respond to urge to void. CONTRIBUTING FACTORS Physical disability or impaired mobility, neurologic disorders - Diuretic therapy or sedation - Lack of facilities, privacy, or caregiver assistance

Types of Urinary Incontinence Reflex

ΤΥΡΕ Reflex DESCRIPTION Involuntary loss of moderate amount of urine occurring without warning CAUSE:Altered spinal cord activity causes hyperreflexia of detrusor muscle Neurologic disorders or trauma SYMPTOMS Voids at regular intervals automatically INTERVENTIONS PATHOPHYSIOLOGY Altered spinal cord activity causes hyperreflexia of the detrusor muscle. CONTRIBUTING FACTORS Neurologic disorders or trauma

Types of Incontinence STRESS

ΤΥΡΕ Stress DESCRIPTION : Loss of small amounts of urine when intra-abdominal pressure increases (sneezing, laughing, lifting).Involuntary loss of small amounts of urine during physical activity that causes increased abdominal pressure cAUSE: Pelvic muscle floor relaxation Weak urethra Multiple pregnancies Decreased estrogen Prostate surgery PATHOPHYSIOLOGY: Pelvic muscle relaxation; weakness of urethra and surrounding tissues decreases urethral resistance. Symptoms:Occurs with laugh, cough, sneeze, or lifting CONTRIBUTING FACTORS - Multiple pregnancies - Decreased estrogen levels (menopause) - Prostate surgery Interventions Provide patient with absorbent pads or panty liners (With stress incontinence, the patient can dribble or leak urine when laughing, coughing...Using a liner will or pad will prevent soiling of clothes.) Exercises: 1. pelvic floor-strengthening exercises (Kegel exercises).-These exercises involve tightening the muscles of the pelvic floor and need to be done numerous times throughout the day. 2.Suggest starting and stopping stream two or three times during voiding to isolate muscles involved in voiding process for exercise training. 3. Incorporate bent-knee sit-ups into exercise program to increase abdominal muscle tone. Meds: Administer medications, as indicated, such as tolterodine (Detrol), oxybutynin (Ditropan), fesoterodine (Toviaz), darifenacin (Enablex), solifenacin (VESIcare), trospium (Sanctura). Medication may improve bladder tone and capacity and increase effectiveness of bladder sphincter and proximal urethra contractions.

Types of Incontinence Urge, overactive bladder

ΤΥΡΕ Urge, overactive bladder DESCRIPTION Involuntary urine loss associated with a strong need to void. Involuntary loss of urine after a strong, abrupt urge to urinate, can't make it to toilet in time CAUSE: -Overactive detrusor muscle→voiding uninhibited -Decreased bladder capacity -Bladder irritation -CNS Disorders SYMPTOMS:Sudden, immediate urge to urinate "gotta go right now" PATHOPHYSIOLOGY Overactive detrusor muscle increases bladder pressure causing inability to inhibit voiding. CONTRIBUTING FACTORS - Decreased bladder capacity - Bladder irritation - Central nervous system disorders INTERVENTIONS: Diet:Adjust fluid intake to 1,500 to 2,000 mL/day if client is prone to ingesting too much fluid. Regulate liquid intake at prescheduled times (with and between meals) and limit fluids 2 to 3 hr prior to bedtime to promote predictable voiding pattern and limit nocturia. Modify foods and fluids as indicated (e.g., reduce caffeine, citrus juices, spicy foods) to reduce bladder irritation. Meds :antibiotic for urinary tract infection, or antimuscarinics [oxybutynin (Ditropan), tolterodine (Detrol), solifenacin (Vesicare)]) to reduce voiding frequency and urgency by blocking overactive detrusor contractions. Exercise: Encourage regular pelvic floor strengthening exercises or Kegel exercises as indicated by specific condition. Instruct the client to tighten pelvic floor muscles before arising from bed. This helps prevent loss of urine as abdominal pressure changes. Suggest starting and stopping stream two or more times during voiding to isolate muscles involved in voiding process for exercise training.

Types of Incontinence Voiding difficulties

ΤΥΡΕ Voiding difficulties DESCRIPTION Urinary retention with bladder overdistention and frequent loss of small amounts of urine (25-50 mL) DEFINITION CAUSE SYMPTOMS INTERVENTIONS PATHOPHYSIOLOGY Outlet obstruction or impaired detrusor muscle activity leads to overfilling and increased pressure. CONTRIBUTING FACTORS - Neurologic disorders or trauma - Enlarged prostate - Fecal impaction - Anticholinergic drug effects

Urinary Calculi What happens to urine out flow with the presence of a stone?

​urine retention leads to increased pressure and distention of the urinary tract behind the obstruction. Hydronephrosis and hydroureter (distention of the ureter with urine) may result. Impaired urine excretion also increases the risk of infection.

MEDS: (tamsulosin)

= pain relief! Thiazide diuretics after analysis for Ca+ Increase fluid intake Dietary restrictions: No peanut M&Ms, beets, spinach, nuts, tea, chocolate, low-sodium Organ meats, sardines, high-purine foods eliminated with stones containing uric acid

Which individual is at high risk for a cobalamin (vitamin B12) deficiency anemia?

A 47-year-old man who had a gastrectomy (removal of the stomach) There are many causes of cobalamin deficiency. The most common cause is pernicious anemia, a disease in which the gastric mucosa is not secreting intrinsic factor (IF) because of antibodies being directed against the gastric parietal cells or IF itself. Other causes of cobalamin deficiency include gastrectomy, gastritis, nutritional deficiency, chronic alcoholism, and hereditary enzymatic defects of cobalamin use.

How does a 3 way irrigation foley work in someone who just had a transurethral resection of the prostate (TURP)? What should the urine look like after surgery?

A TURP is a transurethral resection of the prostate which is usually done in men with symptomatic benign prostatic hypertrophy (BPH). In a TURP, obstructing prostate tissue is removed using the wire loop of a resectoscope inserted through the urethra. (Burke p.799) A three way irrigation foley helps carry resected tissue into the bladder to be flushed out. One port of the three way foley is attached to a large bag of 0.9% sodium chloride that is continuously irrigated through the bladder. Another port is emptying into a foley bag. Light red to red urine with small clots is expected for up to 24 hours after surgery. The urine should gradually clear of clots and become light pink to yellow after 24 to 48 hours. The 3 way irrigation foley consist of 3 lumens: for inflating the balloon which retains the catheter in the bladder, urine drainage and irrigation. The irrigation solution flows into the bladder through the irrigation port of the catheter and out through the urinary drainage lumen of the catheter; carries resected tissue into the bladder to be flushed out on completion of the procedure. Light red to red urine with smalls clots is expected for up to 24 hours after surgery. The urine should gradually clear of clots and become light pink to yellow after 24 to 48 hours . Report (first 24-48 hours): Hemorrhage (frankly bloody urine, large blood clots, decreased urine output, increasing bladder spasms).

Types of Urinary Mixed Incontinence

A combination of stress and urge incontinence is common, known as mixed incontinence Urge, overactive bladder: Could also be caused by caffeine, citrus juices, alcohol and artificially sweetened Tips for functional: Assist, answer call lights quickly, monitor type of clothing (elastic waist). Give diuretics in the AM. Clear pathways, offer bed pan and bedside commodes

Anemia What is it?

A condition i which the hemoglobin concentration or the number of circulating RBCS is decreased o Can also occur if the RBCS don't contain enough hemoglobin. Hemoglobin is an iron-riched protein that gives blood its red color. This protein helps RBCS carry oxygen from the lungs to rest of the body.

Why are patients with peritoneal dialysis (PD) at risk for hyperglycemia and weight gain? What are the nursing interventions to reduce these risks?

A major complication is peritonitis or abdominal infection as all of the fluid exchange is done in the peritoneal cavity. Weight gain and hyperglycemia are common because calories and sugars are absorbed from the dialysate that is used. The dialysate absorbs metabolic waste and electrolytes while in the abdomen and is drained by gravity into a sterile bag. Another factor affecting body image, is the indwelling peritoneal catheter. It is important to note that peritoneal dialysis is less expensive, but is used less often. Nursing interventions to reduce weight gain and hyperglycemia for those undergoing peritoneal dialysis include: accurately monitoring I/O, monitoring daily weight, measure/record abdominal girth, assess for signs of excess fluid volume/edema, assess for s/s of infection (fever, malaise, cloudy dialysate, abdominal pain), document clarity/amount of dialysate returned, restrict fluids as ordered, refer to dietitian consult, time meals with solution drainage, giving small meals (low in sugar, low in protein, low in calories; if there is a prescribed diet, follow that).

When reviewing lab results of an 83 y/o patient with an infection, the nurse would expect to find:

A. minimal leukocytosis

Chronic Kidney Disease: Meds:

ACE inhibitors / ARBS: to decrease proteinuria and blood pressure; diuretics to reduce blood volume and K+/ Sodium bicarb or Calcium Carbonate to manage electrolytes & acidosis o may need *Kayexalate to bind with K+ to decrease level. o CKD: Folic acid and Iron supplements for anemia / Epoetin / MVI / statin **Avoid all nephrotoxic drugs** Patients with CKD have issues with erythropoietin. They will be anemic and need supplementation with folic acid to boost their Hgb cells.


Ensembles d'études connexes

Section 5, Unit 2: Secondary Mortgage Market Concepts

View Set

Mood Adjustment & Dementia-NCLEX 3000

View Set

Bio1107 Exam 2 Possible Questions

View Set

Relations with England Quiz 1: unit 2

View Set